[obm-l] Soma surpreendentemente inteira

2020-03-11 Por tôpico Pedro Cardoso
Olá, amigos.

Gostaria de ajuda para calcular a segunte soma:

Soma com n variando de 1 a 7 de
3/(cos(24n)-1)

Com o argumento do cos em graus

Aparentemente essa soma é 56, não consegui entender porque

-- 
Esta mensagem foi verificada pelo sistema de antiv�rus e
 acredita-se estar livre de perigo.



[obm-l] Re: [obm-l] Ângulos em um triângulo

2020-02-13 Por tôpico Pedro Cardoso
Deve haver um jeito mais fácil, mas foi o que eu pensei agora

Construa os circumcírculos de ABM e NBC. Pela lei dos senos, eles têm o
mesmo raio.
Seja X o centro do circuncírculo de ABM, e Y o de NBC.
B está na intersersão dos circumcírculos, então B está na mediatriz de XY.
AXM, NYC e XBY são isósceles.
ABC e MBN são isósceles
O pé da altura de B em relação a MN coincide com Q.
BQC=90°

Em qui, 13 de fev de 2020 22:42, Vanderlei Nemitz 
escreveu:

> Boa noite!
>
> Usei várias leis dos senos, obtive coisas legais, mas não o ângulo pedido.
> Alguém conhece algo interessante?
>
>
>
> Muito obrigado!
>
>
>
> *Em um triângulo ABC, os pontos consecutivos M, Q, N do lado AC são tais
> que AM = NC. Se Q é ponto médio de MN e os ângulos NBC e ABM medem 20º,
> calcule a medida do ângulo BQC.*
>
> --
> Esta mensagem foi verificada pelo sistema de antivírus e
> acredita-se estar livre de perigo.

-- 
Esta mensagem foi verificada pelo sistema de antiv�rus e
 acredita-se estar livre de perigo.



[obm-l] Re: [obm-l] Re: [obm-l] Máximo

2020-01-12 Por tôpico Pedro Cardoso
O problema é resolvível no contexto do ensino médio porque uma das equações
vão ser retas.
Talvez tenha um jeito ainda mais fácil de resolver, mas essa foi a solução
que encontrei:

Por √x ser crescente, o máximo de
√(16a² + 4b² - 16ab - 12a + 6b + 9)
é a raíz do máximo de
16a² + 4b² - 16ab - 12a + 6b + 9.
Seja
16a² + 4b² - 16ab - 12a + 6b + 9  = p
Então
(4a-2b)² - 3(4a-2b) + 9 = p
(4a-2b)² - 3(4a-2b) + 9/4 = p - 27/4
(4a-2b - 3/2)² = p -27/4
O que dá duas retas:
2a-b = (√(p -27/4) + 3/2 )/2 e
2a-b = (-√(p -27/4) + 3/2 )/2
E queremos encontrar o maior valor de p tal que pelo menos uma dessas retas
toque a equação dada.

Para simplificar, recomendo por
k =  (√(p -27/4) + 3/2 )/2   e
k' =   (-√(p -27/4) + 3/2 )/2

Dessa forma, se encontrarmos o maior valor de k tal que 2a-b = k toca a
equação dada, podemos encontrar também o maior valor de p tal que a
primeira reta toca a equação dada.
Analogamente, se encontrarmos o *menor* valor de *k'* tal que 2a - b = k'
toca a equação, encontramos o maior valor de p tal que a *segunda* reta
toca a equação dada. Aqui fica invertido pois tem um "-" na raíz com o p.

A partir daqui já é possível resolver substituindo 2a-b = k restrição e
encontrando o vértice de uma equação do segundo grau, mas para continuar,
vou aplicar a substituição

a=x+y
b=x-y

temos que
17(a²+b²) -30ab - 16 = 0 fica
34(x²+y²)-30(x²-y²)-16 = 0
4x² + 64y² - 16 = 0
x² + 16y² - 4 = 0

E 2a - b = k fica
2(x + y) - (x - y) = k
x + 3y = k
x = k - 3y

Substituindo, temos
(k-3y)² + 16y² - 4 = 0
k² - 6ky + 25y² - 4 = 0
25y² - 6ky + k²-4 = 0
Essa quadrática em y tem discriminante
Δ = 36k² - 25(4k² - 16)
Δ = 36k² - 100k² + 400
Δ = 400 - 64k²

Quando  Δ>0, a reta toca a equação em mais de 1 ponto. Quando  Δ=0, a reta
é tangente à equação (isso ocorrerá quando k for máximo ou mínimo), e
quando  Δ<0, a reta não toca a equação.

Pondo  Δ=0, temos
25 - 4k² = 0
k =  5/2  e
k = -5/2

Assim, encontramos o maior valor para k:  k = √(25/4)
e o menor valor para k':  k' = -√(25/4). Isso nos dá as equações

5 = √(p -27/4) + 3/2e
-5 = -√(p -27/4) + 3/2
Que não são difíceis de resolver e dão, respectivamente
p = 19   e
p = 49

Como disse no começo, o resultado é a raíz desse máximo, ou seja, √49 = *7*.
Como o resultado foi muito bonito, suspeito que há um jeito muito mais
simples de resolver, mas não o encontrei.
A saber, esse máximo ocorre quando
a = -1,9
b = -1,3

Espero que tenha sido útil
Pedro Cardoso

Em dom., 12 de jan. de 2020 às 18:09, Claudio Buffara <
claudio.buff...@gmail.com> escreveu:

> Oi, Gilberto:
>
> Que mal eu pergunte, de onde veio este problema?
> E por que um aluno de EM teria que resolver um problema desses (e sem usar
> cálculo)?
>
> []s,
> Claudio.
>
>
> On Sun, Jan 12, 2020 at 6:33 PM gilberto azevedo 
> wrote:
>
>> Se a e b são números que satisfazem a equação :
>> 17(a²+b²) - 30ab - 16 = 0
>> Determinar o máximo de :
>> √(16a² + 4b² - 16ab - 12a + 6b + 9)
>> Sem utilizar lagrange e nada que envolva ensino superior . Não sei oq
>> utilizar, se a sacada é enxergar uma fatoração... Enfim aceitando ideias.
>>
>> --
>> Esta mensagem foi verificada pelo sistema de antivírus e
>> acredita-se estar livre de perigo.
>
>
> --
> Esta mensagem foi verificada pelo sistema de antivírus e
> acredita-se estar livre de perigo.

-- 
Esta mensagem foi verificada pelo sistema de antiv�rus e
 acredita-se estar livre de perigo.



[obm-l] Re: [obm-l] Re: [obm-l] Re: [obm-l] Potência

2020-01-11 Por tôpico Pedro Cardoso
Vamos analisar 2^222 módulo 4 e módulo 25. Caso vc não seja familiar a
isso, dizer a = b (mod c) significa dizer que a e b tem o mesmo resto na
divisão por c.

2^222 = 0 (mod 4)

2^222 = 4^111 = (5-1)^111
Expandindo usando o binômio de newton, todos os termos são divisíveis por
25, exceto os dois últimos: (5^1)(1^110) - (5^0)(1^111) =
= 5 - 1 = 4
Ou seja, 2^222 = 4 (mod 25)

04 = 0 (mod 4) e 04 = 4 (mod 25)

Então os últimos dígitos são 04

-- 
Esta mensagem foi verificada pelo sistema de antiv�rus e
 acredita-se estar livre de perigo.



Re: [obm-l] Funcional equation

2019-12-12 Por tôpico Pedro Cardoso
Pensei mais um pouco sobre o problema e acho que encontrei uma solução:

1. Todo polinômio que satisfaz a equação, exceto P(x)=x, tem apenas termos
com expoente par:
Se P(x) tem um termo de grau ímpar, digamos ax^n, podemos escrever P(x) =
ax^n + Q(x) + c, onde c é uma constante diferente de 0 (já que c=0
implicaria P(x)=x, como mostrei no meu outro e-mail)
Escrevemos então P(x^2+1)=(ax^n + Q(x) + c)^2+1
O lado esquerdo tem apenas termos com potências pares. O lado direito pode
ser escrito como (a^2)x^(2n) + (Q(x))^2 + c^2 + 2cQ(x) + 2a(x^n)Q(x) +
2acx^n, que tem termo com grau ímpar 2acx^n, contradição.

2. Se P(x) satisfaz a equação, então Q(x)=P(sqrt(x-1)) também satisfaz:
Temos P(x^2+1)=(P(x))^2+1. Pondo u=x^2+1, temos
P(u)=(P(sqrt(u-1)))^2+1, ou P(u)=(Q(u))^2+1. Mas u=sqrt((u^2+1)-1), então
P(u)=Q(u^2+1). Finalmente
Q(u^2+1)=(Q(u))^2+1
Note que Q também é um polinômio: Já que P só pode ter termos com expoentes
pares, P(sqrt(u-1)) vai cancelar as raízes.

3. O grau de P(x) deve ser uma potência de 2:
Suponha que o grau de P(x) seja k2^n, onde k é ímpar.
Aplique o lema anterior n vezes para obter um polinômio Q(x) de grau k. Mas
soluções da equação funcional não podem ter termos de grau ímpar.
Contradição.

Finalmente, o meu último e-mail mostra que se f(x)=x^2+1, então x, e
f^n(x), onde f^n é a iteração de f n vezes, todos satisfazem a equação, e
f^n(x) é um polinômio de grau 2^n.
Para concluir a solução, o @Esdras Muniz  pode
compartilhar a demonstração de que existe apenas uma solução por grau.

-- 
Esta mensagem foi verificada pelo sistema de antiv�rus e
 acredita-se estar livre de perigo.



Re: [obm-l] Funcional equation

2019-12-09 Por tôpico Pedro Cardoso
Minha intuição foi a seguinte, considere a sequência a_0=0 e
a_(n+1)=(a_n)²+1

Agora pomos P(0)=c
Pela equação funcional, P(0²+1)=c²+1
E P((0²+1)²+1)=(c²+1)²+1

Em geral, se f(x)=x²+1, então

P(a_n)=fⁿ(c), em que fⁿ é a iteração de f n vezes.

Assim, se c=a_n para algum m natural, então vamos ter

P(a_0)=a_n
P(a_1)=a_(n+1)

E em geral, P(x)=fⁿ(x)

Ainda não consegui provar que P não vai ser um polinômio se c não for algum
a_n

Em seg, 9 de dez de 2019 22:43, Pedro Cardoso 
escreveu:

> Preciso pensar mais, mas suspeito que seja qualquer polinômio do tipo
>
> (...((x²+1)²+1)²...)²+1
>
> Os primeiros são
>
> x
> x²+1
> x⁴+2x²+2
> ...
>

-- 
Esta mensagem foi verificada pelo sistema de antiv�rus e
 acredita-se estar livre de perigo.



Re: [obm-l] Funcional equation

2019-12-09 Por tôpico Pedro Cardoso
Preciso pensar mais, mas suspeito que seja qualquer polinômio do tipo

(...((x²+1)²+1)²...)²+1

Os primeiros são

x
x²+1
x⁴+2x²+2
...

-- 
Esta mensagem foi verificada pelo sistema de antiv�rus e
 acredita-se estar livre de perigo.



[obm-l] Re: [obm-l] Teoria dos números

2019-02-05 Por tôpico Pedro Cardoso
Ah, a soma fica sendo 8n-1+1n+1, ou seja, 9n

Em ter, 5 de fev de 2019 15:33, matematica10complicada <
profdouglaso.del...@gmail.com> escreveu:

> Amigos preciso de uma ajuda.
>
> PROBLEMA:
>
> Determinar a soma dos algarismos do produto (888...888)×(999...999), onde
> cada parcela possui "n" algarismos.
>
> Douglas Oliveira.
>
> --
> Esta mensagem foi verificada pelo sistema de antivírus e
> acredita-se estar livre de perigo.

-- 
Esta mensagem foi verificada pelo sistema de antiv�rus e
 acredita-se estar livre de perigo.



[obm-l] Re: [obm-l] Teoria dos números

2019-02-05 Por tôpico Pedro Cardoso
Basta substituir (999...999) por (10^n-1)
O produto será 888...888000...000-888...888, ou seja, 888...887111...112

Em ter, 5 de fev de 2019 15:33, matematica10complicada <
profdouglaso.del...@gmail.com> escreveu:

> Amigos preciso de uma ajuda.
>
> PROBLEMA:
>
> Determinar a soma dos algarismos do produto (888...888)×(999...999), onde
> cada parcela possui "n" algarismos.
>
> Douglas Oliveira.
>
> --
> Esta mensagem foi verificada pelo sistema de antivírus e
> acredita-se estar livre de perigo.

-- 
Esta mensagem foi verificada pelo sistema de antiv�rus e
 acredita-se estar livre de perigo.



[obm-l] Re: [obm-l] relação de girard

2019-02-01 Por tôpico Pedro Cardoso
Expandindo o produto (x-r_1)(x-r_2)(x-r_3)...(x-r_n), ele equivale ao
polinômio x^n-(r_1+r_2+...+r_n)x^(n-1)+...+(-1)^n(r_1r_2r_3...r_n).
Evidentemente, pelo modo como o construímos, esse polinômio tem raízes r_1,
r_2, r_3, ..., r_n. Não é muito difícil ver que a razão entre dois
polinômios com as mesmas raízes é constante. Dessa forma todos os
polinômios com essas raízes podem ser escritos como
ax^n-a(r_1+r_2+...+r_n)x^(n-1)+...+a(-1)^n(r_1r_2r_3...r_n), em que a é um
número real. Isso só funciona se você souber que n raízes existem. Agora,
para garantir que sempre vão existir n raízes num polinômio de enésimo
grau, não vejo outro jeito senão algo equivalente ao TFA

Em sex, 1 de fev de 2019 às 08:38, Israel Meireles Chrisostomo <
israelmchrisost...@gmail.com> escreveu:

> Alguém ai sabe como provar as relações de Girard sem usar o TFA(teorema
> fundamental da álgebra)?
>
> --
> Israel Meireles Chrisostomo
>
> --
> Esta mensagem foi verificada pelo sistema de antivírus e
> acredita-se estar livre de perigo.

-- 
Esta mensagem foi verificada pelo sistema de antiv�rus e
 acredita-se estar livre de perigo.



Re: [obm-l] Amigos comuns (um probleminha)

2017-08-01 Por tôpico Pedro Cardoso
6 pessoas:
Imaginando grafos, vou chamar um trio de pessoas de um triangulo.
1. Note que em um determinado grupo que satisfaz uma das condições, se todas as 
relações entre as pessoas se “inverterem” (ou seja, pessoas que se conhecem 
passam a não se conhecer e vice versa), agora o grupo passa a satisfazer a 
outra condição, afinal um “triângulo” em que 3 pessoas se conhecem vira um em 
que 3 pessoas não se conhecem

2. Se as condições não são satisfeitas e a pessoa A conhece B, C, e D, no 
triangulo ABC, AB se conhecem, e AC se conhecem, então BC não podem se 
conhecer, analogamente, CD, e BD também não podem, mas então existiria o 
triangulo BCD, em que ninguém se conhece, satisfazendo uma condição, logo:
Se uma pessoa conhece 3 outras, a condição obrigatoriamente é satisfeita

3. Consideremos um grupo de 6 pessoas, A,B,C,D,E e F, Agora, vamos analisar a 
relação da pessoa A com todas as outras, uma relação só pode ser Conheçe, ou 
Não conhece, que representarei com C e N,  A relação de A com BCDE, 
respectivamente, pode ter 2 C e 2 N (se não for isso, a condição já estaria 
satisfeita apenas analisando essas 4 relações), mas a relação de A com a pessoa 
F precisa obrigatoriamente ser C ou N, fazendo com que A conheça 3 outras 
pessoas, ou não conheça 3 outras pessoas (no segundo caso, basta “inverter” 
todas as relações e o lema no segundo ponto se aplica), assim, em um grupo com 
6 pessoas, precisa existir alguém que conhece outras 3 pessoas simultaneamente, 
então, aplicando o lema no segundo ponto, precisa existir um triangulo de 
pessoas que não se conhece entre si (ou ao contrario, como foi discutido no 
primeiro ponto).

Finalmente
4. Existem grupos de 5 pessoas em que as condições não se satisfazem:
para isso, basta mostrar um exemplo, considere o grupo de pessoas A,B,C,D e E e 
suponha que todas as arestas no pentágono ABCDE sejam relações “Conhece” e que 
todas as arestas no pentágono ACEBD sejam relações “Não conhece”, aqui as 
condições não são satisfeitas (oara ver isso mais facilmente, basta imaginar 
ABCDE como um pentágono regular e ACEBD como a estrela que se forma dentro dele)

Um Abraço,
Pedro Cardoso 


De: Pedro Chaves
Enviado:terça-feira, 1 de agosto de 2017 17:43
Para: obm-l@mat.puc-rio.br
Assunto: [obm-l] Amigos comuns (um probleminha)

Caros Colegas,
 
 Solicito ajuda para a questão abaixo. 
Abraços do Pedro Chaves. 

--- Amigos comuns ---
Helena é uma perfeita anfitriã. Quando organiza uma festa, se assegura de que 
ao menos três pessoas se conheçam entre si. 
Ou, se isso não for possível, que ao menos haja três pessoas que não se 
conheçam (para assim poder apresentá-las).
Qual é o menor número de pessoas que Helena precisa convidar, para assegurar-se 
de que se dê alguma dessas duas condições?
--xxx

-- 
Esta mensagem foi verificada pelo sistema de antivírus e 
acredita-se estar livre de perigo. 


-- 
Esta mensagem foi verificada pelo sistema de antiv�rus e
 acredita-se estar livre de perigo.



[obm-l] Re: [obm-l]Re: [obm-l] Re: [obm-l] Re: [obm-l] Questão de teoria numérica

2017-08-01 Por tôpico Pedro Cardoso
Realmente. Se isso serve de desculpa eu escrevi isso assim que acordei.
O que eu quis dizer é que não existem múltiplos de 2017 que terminem em 0 e 
que, ao serem divididos por 10, deixam de ser múltiplos de 2017. Para isso 
existir, 2017 teria que ter um número de fatores 2 diferente do número de 
fatores 5, mostrar porque isso não é difícil: supondo que k tem x fatores 2 e y 
fatores 5 se x>y temos que 5k/10 tem x-1 fatores 2 e y fatores 5, e portanto, 
não é divisível por k, para x escreveu:
>
> Obrigado! Era exatamente isso que a questão anterior sugeria, usar o 
> princípio da casa dos pombos.
> Uma coisa que percebi na sua dsmonstração é que o número encontrado 
> terminaria em 0s, mas como nenhum multiplo de 2017 também é multiplo de 10 
> (2017 é primo) então também existe um multiplo de 2017 com apenas 1s!

A conclusão está certa (existe um múltiplo de 2017 só com 1s), mas a
justificativa está errada: 20170 é múltiplo de 2017 e de 10.  Tem a
ver com primalidade, mas não é bem o que você escreveu.

Abraços,
-- 
Bernardo Freitas Paulo da Costa

-- 
Esta mensagem foi verificada pelo sistema de antiv�rus e
 acredita-se estar livre de perigo.


=
Instru��es para entrar na lista, sair da lista e usar a lista em
http://www.mat.puc-rio.br/~obmlistas/obm-l.html
=


-- 
Esta mensagem foi verificada pelo sistema de antiv�rus e
 acredita-se estar livre de perigo.



[obm-l] Re: [obm-l] Re: [obm-l] Questão de teoria numérica

2017-08-01 Por tôpico Pedro Cardoso
Obrigado! Era exatamente isso que a questão anterior sugeria, usar o
princípio da casa dos pombos.
Uma coisa que percebi na sua dsmonstração é que o número encontrado
terminaria em 0s, mas como nenhum multiplo de 2017 também é multiplo de 10
(2017 é primo) então também existe um multiplo de 2017 com apenas 1s!

Obrigado.
Pedro Cardoso

Em 31 de jul de 2017 23:31, "Adilson Francisco da Silva" <
adilson...@gmail.com> escreveu:

> Salve!
>
> Construa uma sequência com 2018 números naturais da seguinte forma:
> 1
> 11
> 111
> 
> .
> .
> .
> 111...1 (2018 dígitos 1).
>
> Pelo princípio da casa dos pombos existe ao menos dois desses números que
> deixam o mesmo resto na divisão por 2017.
>
> Use o fato de que se dois números deixar o mesmo resto na divisão por um
> certo número d, então a diferença entre eles é divisível por d.
>
> Assim pegue os dois termos da sequência que deixa o mesmo resto é faça
>
> 11...111 - 11...1 = 111...100...0
>
> Que é divisível por 2017.
>
> Abraços
>
>
> Em 31 de jul de 2017 10:38 PM, "Pedro Cardoso" <mr.pedrocard...@gmail.com>
> escreveu:
>
> Segue uma questão de teoria numérica da Olimpíada SESI de Matemática (AM):
>
> Mostre que existe um múltiplo de 2017 formado apenas pelos dígitos 0 e 1
> (em base 10).
>
>
>
> Na olimpíada, a questão anterior sugere uma maneira de resolver, porém,
> estou interessado em outras demonstrações também.
>
> Se ninguém conseguir achar uma prova, mando a outra questão mais tarde.
>
> Att.
> Pedro Cardoso
>
>
>
> --
> Esta mensagem foi verificada pelo sistema de antivírus e
> acredita-se estar livre de perigo.
>
>
>
> --
> Esta mensagem foi verificada pelo sistema de antivírus e
> acredita-se estar livre de perigo.

-- 
Esta mensagem foi verificada pelo sistema de antiv�rus e
 acredita-se estar livre de perigo.



[obm-l] Questão de teoria numérica

2017-07-31 Por tôpico Pedro Cardoso
Segue uma questão de teoria numérica da Olimpíada SESI de Matemática (AM):

Mostre que existe um múltiplo de 2017 formado apenas pelos dígitos 0 e 1 (em 
base 10).

Na olimpíada, a questão anterior sugere uma maneira de resolver, porém, estou 
interessado em outras demonstrações também. 
Se ninguém conseguir achar uma prova, mando a outra questão mais tarde.
Att.
Pedro Cardoso


-- 
Esta mensagem foi verificada pelo sistema de antiv�rus e
 acredita-se estar livre de perigo.



[obm-l] Re: [obm-l] FW: Múltiplos

2011-12-06 Por tôpico Pedro Cardoso
Oi, João.

Na decomposição por números primos, o número de 2 que aparece em n! é...

S = n/2 + n/4 + n/8 + ... + 1 = n * [ 1/2*(1 - 1/2^T)/(1-1/2) ]
onde T = número de termos da PG. Veja que n * (1/2)^T = 1.

Pense um pouco pra ver que isso vale pra qualquer primo (o +1 não, o +1 no
fim eu só posso colocar porque n é potência de 2 pras contas fecharem).

Daí,

S = n * [ 1/2*( 1 - 1/n) / (1/2) ] = n*(1-1/n) = n-1.

Assim, 2 aparece n-1 vezes em n! e portanto 2^(n-1) divide n! (isso
tudo quando n é potência de 2, pro somatório ficar bonito).

2011/12/6 João Maldonado joao_maldona...@hotmail.com



 --
 From: joao_maldona...@hotmail.com
 To: obm-l@mat.puc-rio.br
 Subject: Múltiplos
 Date: Tue, 6 Dec 2011 14:29:31 -0200



 Prove que se n é potênica de 2,  então n! é múltiplo de 2^(n-1)

 O jeito que eu fiz  é um pouquinho complicado,  consiste  principalmente
 em calcular o somatório (n-p).2^(p-1),  com p variando de 1 a n-1
 Deu certo mas  foi somente contas, existe algum jeito de pensar mais
 bonitinho   (até porque no contexto que vi esse problema não deve ser essa
 a solução mais fácil)?

 []'s
 João



[obm-l] Re: [obm-l] Re: [obm-l] Um problema curioso e... insolúvel

2011-05-20 Por tôpico Pedro Cardoso
Opa, Bruno, o processo que você descreveu certamente faz o nadador achar uma
das margens.
Mas o Bouskela quer mais - ele quer saber a melhor maneira (que faz o
nadador nadar menos)
de se achar uma das margens.

Acho que isso cai para uma área da matemática que os matemáticos puros não
estudam muito
- a área de algoritmos. E esse problema tem a maior cara de *busca
exponencial*.

---

Imagine que o nadador está a uma distância N de uma das margens.
Ele deve fazer o seguinte...

x - 1
Enquanto não achar a margem, repita:
[1] Nada x metros pra frente. Volta. Nada 2x metros para trás. Volta.
[2] Nada x metros pra esquerda. Volta. Nada 2x metros pra direita. Volta.
x -  4x

As noções de frente e esquerda estão erradas no máximo 45 graus. Assim,
na pior das hipóteses o nadador vai ter que nadar sqrt(2)*N metros em uma
das direções
(agora, até fazer isso, ele nadou nessa direção várias vezes).

Note que [1] e [2] são processos independentes.

Quanto ele nada em função de N? Pense um pouco pra ver que é C*N, onde C é
uma constante.

Ignorando constantes, essa é a melhor maneira, já que mesno enxergando ele
teria que nadar 1*N
metros.

2011/5/19 Bruno França dos Reis bfr...@gmail.com

 Em aberto?

 Se o nadador estivesse nadando paralelo ao rio, é só ele fazer uma curva
 mínima, e continuar até chegar às margens.

 Caso o nadador não saiba a direção em que estava nadando (suponhamos uma
 briga com os peixes, que o deixou desorientado, antes de ter seus olhos
 devorados), ele poderia nadar seguindo uma espiral, aí certamente
 encontrará a margem, não? O algoritmo seria:

 n - 1
 Enquanto não achar a margem, repita:
  - dar n braçadas para frente
  - virar 90 graus para a esquerda
  - dar n braçacas para frente
  - virar 90 graus para a esquerda
  - n - n + 1

 Como a largura é finita, e a espiral cresce de tamanho em todas as
 direções, esse algoritmo certamente termina em um tempo finito!

 Tem alguma falha que eu não vi nesse processo?

 Abraço!
 Bruno


 --
 Bruno FRANÇA DOS REIS

 msn: brunoreis...@hotmail.com
 skype: brunoreis666
 tel: +55 11 9961-7732

 http://brunoreis.com
 http://brunoreis.com/tech (en)
 http://brunoreis.com/blog (pt)

 GPG Key: http://brunoreis.com/bruno-public.key

 e^(pi*i)+1=0



 2011/5/19 Albert Bouskela bousk...@msn.com

 Olá a todos,



 Uma curiosidade: – Parece-me que o problema abaixo (tão simples!)
 permanece em aberto.



 Um nadador está nadando (o que mais pode fazer um nadador?) em um ponto
 qualquer de um rio horizontal, retilíneo, com correnteza desprezível,
 comprimento infinito e largura finita.



 Subitamente, peixes extremamente vorazes devoram os olhos do malfadado
 nadador, ou, com menos drama, cai a noite absolutamente escura.



 Qual é a trajetória que o nadador deve trilhar, i.e., nadar, para atingir
 – seguramente – uma das margens, nadando a menor distância possível?



 Obs.: – O malfadado nadador tem, implantado em sua cabeça, um sistema de
 navegação que lhe informa, continuamente, a sua posição em relação ao ponto
 inicial (o ponto no qual os peixes devoraram os seus olhos).



 Saudações,

 Albert Bouskela

 bousk...@msn.com







[obm-l] Re: [obm-l] Re: [obm-l] Re: [obm-l] Re: [obm-l] Re: [obm-l] Re: [obm-l] Re: [obm-l] Amigos na roda (indução (ou não))

2011-02-28 Por tôpico Pedro Cardoso
Opa, descobri outro dia que a prova desse problema (na versão para grafos),
é um teorema com nome - teorema de Dirac (não o Paul). Não deve sair por
indução mesmo!

2011/2/28 Rogerio Ponce abrlw...@gmail.com

 Oi Bernardo e Pedro,
 voces tem razao!
 Nao da' para usar inducao quando temos 2K pessoas, pois ao tirarmos o Joao,
 talvez nem todos os participantes do grupo 2K-1 permanecam com o minimo de K
 amigos ( teto(2K-1) = K ).
 Portanto, nao podemos aplicar a hipotese ao grupo 2K-1.
 Em provas por inducao, qualquer falta de atencao induz ao erro...
 :)
 Abracao,
 Rogerio Ponce


 Em 25 de fevereiro de 2011 13:48, Bernardo Freitas Paulo da Costa 
 bernardo...@gmail.com escreveu:

 Oi Ponce !

 2011/2/25 Rogerio Ponce abrlw...@gmail.com:
  Bernardo,
  acho que voce se confundiu nisso daqui:
 
  Se você retirar qualquer um dos participantes de grupo, já era, porque
  sobram (sem perda de generalidade) A,B e C, e você não pode botar A do
 lado
  de C...
 
  Nos queremos justamente colocar pessoas lado a lado, e o grupo esta'
 reunido
  numa roda.
 Ah, ok... Mas eu continuo achando que botar as pessoas lado a lado
 não é garantido pela hipótese de indução... Para mim a H.I. é Todo
 grafo de k vértices, todos de grau = k/2, possui um ciclo. Você quer
 um quase-ciclo, e você pede um pouco menos do que grau = k/2. Pode
 ser a mesma coisa, eu só não tenho certeza, e confesso que não tive
 tempo para pensar nisso essa semana. Se for mesmo, eu me desculpo de
 ser tão Bourbakista aqui.

  []'s
  Rogerio Ponce

 Abraços,
 --
 Bernardo Freitas Paulo da Costa

 =
 Instruções para entrar na lista, sair da lista e usar a lista em
 http://www.mat.puc-rio.br/~obmlistas/obm-l.html
 =





-- 
Abraços,
Pedro.


[obm-l] Re: [obm-l] Re: [obm-l] Re: [obm-l] Re: [obm-l] Re: [obm-l] Re: [obm-l] Amigos na roda (indução (ou não))

2011-02-26 Por tôpico Pedro Cardoso
Olá...

Ponce, eu ainda acho que o Bernado está certo - não dá pra usar a H.I.
quando se tira o João da roda.

2011/2/25 Bernardo Freitas Paulo da Costa bernardo...@gmail.com

 Oi Ponce !

 2011/2/25 Rogerio Ponce abrlw...@gmail.com:
  Bernardo,
  acho que voce se confundiu nisso daqui:
 
  Se você retirar qualquer um dos participantes de grupo, já era, porque
  sobram (sem perda de generalidade) A,B e C, e você não pode botar A do
 lado
  de C...
 
  Nos queremos justamente colocar pessoas lado a lado, e o grupo esta'
 reunido
  numa roda.
 Ah, ok... Mas eu continuo achando que botar as pessoas lado a lado
 não é garantido pela hipótese de indução... Para mim a H.I. é Todo
 grafo de k vértices, todos de grau = k/2, possui um ciclo.


Bernardo, um ciclo contendo todos os vértices!


 Você quer
 um quase-ciclo, e você pede um pouco menos do que grau = k/2. Pode
 ser a mesma coisa, eu só não tenho certeza, e confesso que não tive
 tempo para pensar nisso essa semana. Se for mesmo, eu me desculpo de
 ser tão Bourbakista aqui.

  []'s
  Rogerio Ponce

 Abraços,
 --
 Bernardo Freitas Paulo da Costa

 =
 Instruções para entrar na lista, sair da lista e usar a lista em
 http://www.mat.puc-rio.br/~obmlistas/obm-l.html
 =




-- 
Abraços,
Pedro.


[obm-l] Re: [obm-l] Amigos na roda (indução (ou não))

2011-02-24 Por tôpico Pedro Cardoso
Opa, perdão! Confundi teto[ (2N+1)/2] = N+1. De qualquer forma, não funciona
do N ímpar pro N par mesmo. Olha só, Bernardo, Rogério e os demais...

 repetindo o enunciado por questão de clareza ---

Prove que num grupo de N pessoas - onde cada pessoa tem pelo menos
teto(N/2) amigos - é
possível organizar todo mundo numa roda, de modo que cada pessoa fique entre
amigos.


Obs.1: se A é amigo de B, então B é amigo de A.
Obs.2: eu vou chamar a maneira de arrumar as pessoas de modo que elas entre
amigos
de arrumação legal.

-

[I] Hipótese: num grupo de *2K+1* pessoas, cada pessoa tem *K+1* amigos e é
possível fazer
uma arrumação legal.

[II] Num grupo de *2K+2* pessoas, cada pessoa tem *K+1* amigos. Tire o João
do grupo.
Agora restam *2K+1* pessoas e cada uma tem *K* amigos (no mínimo). Por
[I]... OPA! Não
dá pra usar [I].



Bernardo, não consegui ainda ter uma ideia pra achar a probabilidade de um
arranjo
aleatório ser uma arrumação legal. Mas continuo tentando!


[obm-l] [obm-l] Amigos na roda (indução (ou não))

2011-02-23 Por tôpico Pedro Cardoso
Olá.

Meu professor propôs essa questão (retirada de um livro), mas depois de
discutirmos
chegamos à conclusão de que a solução dele não estava certa. Queria saber se
alguém resolve.

Prove que num grupo de N pessoas - onde cada pessoa tem pelo menos
teto(N/2) amigos - é
possível organizar todo mundo numa roda, de modo que cada pessoa fique entre
amigos.

Obs.1: se A é amigo de B, então B é amigo de A.
Obs.2: eu vou chamar a maneira de arrumar as pessoas de modo que elas entre
amigos
de arrumação legal.

Isso pede indução, mas eu não consegui provar. Foi fácil ir de N ímpar pra N
par...

[I] Hipótese: num grupo de 2N+1 pessoas, cada pessoa tem N amigos e é
possível fazer
uma arrumação legal.

[II] Num grupo de 2N+2 pessoas, cada pessoa tem N+1 amigos. Tire o João do
grupo.
Agora restam 2N+1 pessoas e cada uma tem N amigos (no mínimo). Por [I], é
possível
fazer uma arrumação legal com as 2N+1 pessoas restantes. Agora ponha o João
entre duas
pessoas que ele conhece e que estão juntas. Isso é possível pelo princípio
da casa dos pombos,
já que o João conhece N+1 das 2N+1 pessoas da roda.

Agora, quando em [I] a quantidade de pessoas é par, a ideia não vinga.

Abraços,

Pedro Cardoso.


[obm-l] Re: [obm-l] Re: [obm-l] OFF-TOPIC:memorização de números primos

2011-02-13 Por tôpico Pedro Cardoso
Poxa, pra ver ser um numero menor do que 150 é primo basta verificar se ele
é divisível por 2,3,5,7,11. Por 2,3 e 5 é imediato. Então não vale a pena
decorar.

Abraços!

2011/2/13 Zair Henrique warcraftschevei...@gmail.com

 Acho que a prática de exercícios envolvendo os números primos, como
 fatoração já faz com que fique memorizado os números. Ficar tentando decorar
 não adianta.

 Em 13 de fevereiro de 2011 18:37, Silas Gruta silasgr...@gmail.comescreveu:

 Olá mestres,

 alguns alunos me perguntaram se existe uma técnica prática de memorização
 dos números primos, tipo, os menores que 100 ou 150. Sei que esse tipo de
 decoreba não é matemática, mas para quem quer se preparar para concursos
 militares é de certa ajuda.

 um abraço

 --
 Silas Gruta







-- 
Abraços,
Pedro.


Re: [obm-l] Enc: Limite com raizes

2010-08-03 Por tôpico Pedro Cardoso
Oi, Graciliano.

Você não precisa manipular. Basta usar o teorema do confronto.

Sejam...

g(x) = raiz(x)
f(x) = raiz(x+raiz(x+raiz(x)))
h(x) = raiz(3x) [eu só troquei x+raiz(x) por 2x e depois x+raiz(2x) por 3x]

Pra x  1, g = f = h.

2010/8/3 Graciliano Antonio Damazo bissa_dam...@yahoo.com.br



 --- Em *seg, 2/8/10, Graciliano Antonio Damazo bissa_dam...@yahoo.com.br
 * escreveu:


 De: Graciliano Antonio Damazo bissa_dam...@yahoo.com.br
 Assunto: Limite com raizes
 Para: obm-l@mat.puc-rio.br
 Data: Segunda-feira, 2 de Agosto de 2010, 10:57


Galera como posso manipular esse limite:

 lim  raiz(x+raiz(x+raiz(x)))
 x--infinito
 Desde de já agradeço.









-- 
Pedro Lazéra Cardoso
http://aprendiepostei.blogspot.com


[obm-l] Formigas

2010-03-10 Por tôpico Pedro Cardoso
Problema proposto pelo Bernardo (Freitas Paulo da Costa):

Você bota 2000 formigas num círculo de comprimento 1 (ok, as formigas
são pontos sem comprimento). Dispostas como você quiser. Elas andam no
círculo a 1m/s (são formigas turbinadas, o importante é que elas
dariam a volta no círculo em 1s). O único problema é que elas não vão
todas no mesmo sentido. E, obviamente, quando elas se encontram, elas
mudam de direção, sem perder tempo nem velocidade (para os físicos, um
choque elástico perfeito). Prove que 1000 segundos depois, as
formigas estão na mesma posição que você as colocou no início.

-- fim do problema
-

Oi, Bernardo. Mas as formigas são distintas ou trocar duas de posição
não muda nada?


Re: [obm-l] Formigas

2010-03-10 Por tôpico Pedro Cardoso
Oi, Marcelo.

Foi por isso que perguntei se as formigas são distintas: se *não*
forem, aí o que você falou vale mesmo.

Abraços,

Pedro.

2010/3/10 Marcelo Salhab Brogliato msbro...@gmail.com

 Olá Pedro,
 quando as formigas colidirem e mudarem de direção, é exatamente igual a
 elas se atravessarem.
 Como elas dão 1 volta por segundo, 1 segundo dps elas estarão lá, 2
 segundos dps elas estarão lá,
 1000 segundos dps elas estarão lá. ;)

 abraços,
 Salhab


 2010/3/10 Pedro Cardoso pedrolaz...@gmail.com


 Problema proposto pelo Bernardo (Freitas Paulo da Costa):

 Você bota 2000 formigas num círculo de comprimento 1 (ok, as formigas
 são pontos sem comprimento). Dispostas como você quiser. Elas andam no
 círculo a 1m/s (são formigas turbinadas, o importante é que elas
 dariam a volta no círculo em 1s). O único problema é que elas não vão
 todas no mesmo sentido. E, obviamente, quando elas se encontram, elas
 mudam de direção, sem perder tempo nem velocidade (para os físicos, um
 choque elástico perfeito). Prove que 1000 segundos depois, as
 formigas estão na mesma posição que você as colocou no início.

 -- fim do problema
 -

 Oi, Bernardo. Mas as formigas são distintas ou trocar duas de posição
 não muda nada?





[obm-l] Re: [obm-l] RE: [obm-l] Re: [obm-l] Onde está o err o?

2010-02-02 Por tôpico Pedro Cardoso
Suponha,por absurdo,que o maior  número natural fosse um
n1.Então,multiplicando ambos os membros desta desigualdade por n,teríamos
(n^2)  n.Uma contradição pois estamos supondo q n é o maior número
natural.

Artur, não entendi: onde se está assumindo, no raciocínio acima, a hipótese
de que 1 é o maior natural?

Abraços,

Pedro.

2010/2/2 Artur Steiner artur_stei...@hotmail.com

  Eu gostaria de frisar que, na minha opinião, o principal furo é se tentar
 provar uma hipótese partindo do princípio de que a mesma é verdadeira. Isto
 é um sofisma lógico, não pode ser empregado nem mesmo para provar o que é
 verdade. Por exemplo, se n é ímpar, então n^2 = 1 (mod 4). Isto pode ser
 provado, mas considerando-se outras propriedades dos números ímpares. Embora
 a proposição seja verdadeira, não podemos prová-la já supondo que  n^2= 1
 (mod 4). Isto, simplesmente, não é prova.
 Artur

 --
 Date: Tue, 2 Feb 2010 14:58:37 -0200
 Subject: [obm-l] Re: [obm-l] Onde está o erro?
 From: ralp...@gmail.com

 To: obm-l@mat.puc-rio.br

 Vou me ater aa pergunta original, e meio que repetir o que o Lucas jah
 disse, que achei ser a melhor explicacao.

 O seguinte raciocinio estah CORRETO:

 Suponha que o maior número natural fosse um n1.Então,multiplicando ambos
 os membros desta desigualdade por n,teríamos (n^2)  n.Uma contradição pois
 estamos supondo q n é o maior número natural.

 Com isto, voce provou que:

 SE houver um maior numero natural, ENTAO ele tem que ser 1.

 Em outras palavras:

 O unico POSSIVEL maior natural eh 1

 ou

 O maior natural eh 1 ou nao existe

 O unico erro eh concluir entao que 1 eh o maior natural -- para tanto, voce
 teria que provar agora que HA um maior natural... E, como o pessoal disse,
 isto voce nao vai conseguir.

 Abraco, Ralph.

 2010/1/29 marcone augusto araújo borges marconeborge...@hotmail.com

 Onde está o erro na seguinte ´´prova´´ de q 1 é o maior número
 natural:´´Suponha,por absurdo, que o maior  número natural fosse um
 n1.Então,multiplicando ambos os membros desta desigualdade por n,teríamos
 (n^2)  n.Uma contradição pois estamos supondo q n é o maior número
 natural.Eu gostaria de um esclarecimento.Obrigado.
 --
 Quer fazer um álbum íncrivel? Conheça o Windows Live Fotos clicando 
 aqui.http://www.eutenhomaisnowindowslive.com.br/?utm_source=MSN_Hotmailutm_medium=Taglineutm_campaign=InfuseSocial



 --
 Quer fazer a bateria do seu notebook render mais? Clique aqui e descubra
 como.http://www.microsoft.com/brasil/windows7/default.html?WT.mc_id=1539



Re: [obm-l] Algoritmo

2009-09-21 Por tôpico Pedro Cardoso
Oi, Teofilo.
Em C, basta ver se (num1) = 1 (que, no caso, é ver se (num1) == 1 ).
É mais rápido do que div e mod, eu acho.
Pesquise na web o que significa o operador .

Agora, se não vale trapaça, mas valer olhar o sinal do número, então faz,
para números positivos:

vai tirando 2 do número até num = 0.
Se num  0, então é ímpar (foi de 1 para -1)
Se num = 0, então é par. (foi de 2 para 0)

Se num for de cara negativo, rode o algoritmo com (-num).
Mas é um algoritmo horrível!

Abraços,

Pedro.

2009/9/21 Teofilo Viturino professorteof...@gmail.com


 Como construir um algoritmo que determine se um número é par ou impar sem
 usar DIV e MOD? Alguém poderia me ajudar com essa?

 --
 Teófilo Viturino
 (81) 8771-0500





RE: [obm-l] Probabilidade- problema das moedas de Bertrand (adaptado)

2009-07-19 Por tôpico Pedro Cardoso


Sobre o problema C do Ralph, em que eu tentei por todas as informações 
necessáias,
pra quem pegar a conversa no meio não ter que ficar adivinhando algumas
coisas.

PROBLEMA C 

Temos inicialmente 3 caixas, 
caixa 1 com 2 moedas de ouro (O1O2), 
caixa  com 2 moedas de prata (P1P2), 
e caixa 3 com uma moeda de cada (O3P3).

Escolhe-se uma caixa ao acaso, e seleciona-se uma moeda,
que eh reposta na sua caixa (ou não, pois não faz diferença). Novamente, 
escolhe-se uma caixa ao acaso, DIFERENTE da primeira caixa, e retira-se uma 
SEGUNDA moeda. Sabendo que a primeira moeda eh de ouro, qual a chance de a 
segunda ser de ouro tambem?

-

Bom, no caso as possibilidades de se tirar duas moedas de ouro são:

o1o3 = 1/3*1/2 * 1/2*1/2 (escolho caixa 1, escolho o1; escolho caixa 2, escolho 
o3)
o3o1 = 1/3*1/2 * 1/2*1/2 (parecido)
o2o3 = 1/3*1/2 * 1/2*1/2 (parecido)
o3o2 = 1/3*1/2 * 1/2*1/2 (parecido)

Somando tudo, dá 1/3 * 1/8 * 4 = 1/6
Daí, ainda usando as nomenclaturas do Ralph,

P(OO|OX) = 1/6 / 1/2 = 1/3.

Vejam aí se não errei alguma coisa.

Enfim, Ralph, com a sua explicação sobre o PROBLEMA B, acho que ficou 
bem mais fácil. 

Aliás, só para te encorajar a continuar: eu aposto que bastante 
gente lê os e-mails que você escreve, inclusive aqueles em que você diz algo 
como
Talvez ninguém leia isso. Aprendi bastante coisa de combinatória com
você. Obrigado mesmo!


 Date: Wed, 15 Jul 2009 04:19:40 -0300
 Subject: Re: [obm-l] Probabilidade- problema das moedas de Bertrand (adaptado)
 From: ralp...@gmail.com
 To: obm-l@mat.puc-rio.br
 
 Oi, Claudio.
 
 Explica um pouquinho melhor a variacao que voce estah pedindo Digo
 isso porque, no problema original, nao ha uma segunda moeda sendo
 RETIRADA. No original, a pergunta eh se a moeda retirada eh de ouro,
 qual a chance de a outra moeda DESTA MESMA CAIXA ser de ouro tambem?.
 Ela nem retirada eh
 
 Se voce vai retirar uma segunda moeda, tem de explicar COMO a segunda
 retirada eh feita. Entao vejamos: temos inicialmente 3 caixas, caixa 1
 com 2 moedas de ouro (O1O2), caixa  com 2 moedas de prata (P1P2), e
 caixa 3 com uma moeda de cada (O3P3).
 
 PROBLEMA A: Escolhe-se uma caixa ao acaso, e seleciona-se uma moeda,
 que eh reposta na sua caixa. Novamente, escolhe-se uma caixa ao acaso,
 independentemente da primeira escolha, e retira-se uma SEGUNDA moeda.
 Sabendo que a primeira eh de ouro, qual a chance de a segunda ser de
 ouro tambem?
 RESPOSTA: Retiradas independentes, entao a informacao da primeira
 moeda nao diz nada. Resposta 3/6=1/2.
 
 PROBLEMA B: Idem ao A, mas a primeira moeda nao eh reposta.
 RESPOSTA: Fica melhor se desenhar uma arvore com quase 36 ramos...
 Bom: ha 6 maneiras de tirar duas moedas de ouro: O1O2, O2O1, O1O3,
 O2O3, O3O1, O3O2. As duas primeiras somam 1/9 (escolher caixa 1 duas
 vezes); as duas proximas somam 1/3.1/3.1/2 (caixa 1, depois caixa 2,
 moeda O3); e a terceira tem probabilidade 1/3.1/2.1/3. Somando tudo,
 Pr(OO)=2/9.
 Agora, a probabilidade da primeira moeda ser de ouro eh 1/2. Entao, a
 probabilidade pedida eh Pr(OO|OX)=(2/9)/(1/2)=4/9.
 Outra maneira de fazer: a primeira moeda veio da caixa com OO com 2/3
 de chance; neste caso, a chance da segunda ser O eh 1/3+1/3.1/2=1/2
 (na segunda retirada, 1/3 de pegar a mesma caixa, e 1/3 de pegar a
 caixa OP). Se a primeira veio de OP, a segunda soh eh  se voce
 escolher a caixa OO, isto eh, 1/3 de chance. Juntando tudo:
 Pr(OO|OX)=2/3.1/2+1/3.1/3=4/9
 
 PROBLEMA C: A segunda caixa TEM DE SER DIFERENTE DA PRIMEIRA; neste
 caso nao faz diferenca se a primeira moeda eh reposta ou nao Deixo
 esse pra voces. Resposta: 2/3.1/4+1/3.1/2=1/3.
 
 Abraco, Ralph.
 
 2009/7/14 Claudio Dias claudiomd...@hotmail.com:
  Oi, Walter.
 
  O problema original é dessa forma( resposta 2/3). Ele acaba induzindo a
  mesma caixa. Mas se não tivesse que ser da mesma caixa. Explo. a primeira
  retirada era da segunda caixa e a segunda da primeira ou a primeira retirada
  é da caixa 1 e a segunda da caixa 2. Esse foi o questionamento.
 
  
  Date: Tue, 14 Jul 2009 21:22:18 -0300
  Subject: Re: [obm-l] Probabilidade- problema das moedas de Bertrand
  (adaptado)
  From: wtade...@gmail.com
  To: obm-l@mat.puc-rio.br
 
  Oi, Claudio
 
  A pergunta não se resumiria em Se a moeda selecionada é de ouro, qual  a
  probilidade de ser da caixa 1?.
  Tentei fazer a árvore e saiu assim:
 
  Ramo 1: P(cx1).P(ouro) = (1/3). (1) (seleciona a caixa 1 e sempre sai ouro)
  Ramo 2: P(c2).P(ouro) = (1/3).(1/2) (seleciona a caixa 2 e sai um ouro com)
  Ramo 3: P(cx3).P(ouro) = (1/3).(0) (seleciona a caixa 3 e não tem ouro)
 
  P(ouro) = (1/3).(1)+(1/3).(1/2) = 1/3 + 1/6 = 3/6 = 1/2
  P(cx1/ouro) = P(cx1 e ouro)/P(ouro) = (1/3)/(1/2) =2/3
 
  Fiz besteira?
 
  Abraços
 
  2009/7/14 Fabio Bernardo prof_fabioberna...@yahoo.com.br
 
  Vc só esqueceu de postar o problema... Rs...
 
  - Original Message -
  From: Claudio Dias
  To: obm-l@mat.puc-rio.br
 

RE: [obm-l] Probabilidade

2009-07-07 Por tôpico Pedro Cardoso


Oi, Pedro.Seria legal se você explicasse como fez.
Abraços,
Pedro Cardoso.

From: npc1...@gmail.com
To: obm-l@mat.puc-rio.br
Subject: [obm-l] Probabilidade
Date: Mon, 6 Jul 2009 10:00:25 -0300
















Amigos da lista, a resposta será
 P=7,64% ?

 

1)Considere um grupo de quatro estudante do
IFRN. Qual a probabilidade de dois deles fazerem aniversário no mesmo mês e de
os outros dois aniversariarem em outro mesmo mês?

 


_
Deixe suas conversas mais divertidas. Baixe agora mesmo novos emoticons. É 
grátis!
http://specials.br.msn.com/ilovemessenger/pacotes.aspx

RE: [obm-l] Problema Bonito - Probabilidade

2009-05-20 Por tôpico Pedro Cardoso

 

Oi, Fernando. Veja que os pontos pertencentes às retas são as 

'situações limites' - aquelas em que uma pessoa chega exatamente

10min depois (ou antes, né?) da outra. Além disso, um par (a,b), para fazer

parte do conjunto solução do problema, deve satisfazer três coisas:

 

[1] a-b  -10 .:. b  a+10 (isto é, b está abaixo da reta y = a+10)

[2] a-b  10 .:. b  a - 10 (isto é, b está acima da reta y = a-10)

[3] 0 = a,b = 60

 

Então, se (a,b) satisfaz as condições de encontro das duas pessoas,

então (a,b) está entre essas retas E dentro do quadrado. O resto é conta.

 

Abraços,


Pedro Lazéra Cardoso
 


Date: Tue, 19 May 2009 22:59:55 -0300
Subject: Re: [obm-l] Problema Bonito - Probabilidade
From: fgam...@gmail.com
To: obm-l@mat.puc-rio.br


Não entendi porque destas retas:


Desenhar as retas y = x+10 e y = x-10.


A região do quadrado entre as retas (região S) forma o conjunto de pares (a,b) 
tal que abs(a-b) = 10, isto é, os pares
que representam tempos de chegada para os quais há encontro entre as duas 
pessoas. O quadrado todo representa o conjunto de todos os pares possíveis 
(tudo em minuto, claro). Assim, como os pares são equiprováveis...

2009/5/19 Pedro Cardoso pedrolaz...@hotmail.com


Olá. Eu acho que é assim: 


Problema: 


luiz silva escreveu:
Duas pessoas marcam um encontro em um determinado local. Combinam que ambos 
deverão chgegar a este local entre 12 e13 h. Porém, qdo o 1o. chegar ao local, 
irá esperar 10 min pelo outro. Caso o outro não chegeu ao local nete intervalo 
de tempo (10 min), o primeiro a chegar vai embora, e eles não conseguem se 
encontrar. Qual a probabilidade do encontro ocorrer ?


Bom, seja (a,b) o par que representa os dois instantes em que as duas pessoas 
chegaram, onde 0 = a,b = 60.
Como a chance de ocorrência dos pares é igual, vale desenhar um quadrado de 
lado 60 no plano cartesiano,
cujos vértices ficam nos pontos (0,0), (60,0), (60,60), (0,60), e fazer o 
seguinte:


Desenhar as retas y = x+10 e y = x-10.


A região do quadrado entre as retas (região S) forma o conjunto de pares (a,b) 
tal que abs(a-b) = 10, isto é, os pares
que representam tempos de chegada para os quais há encontro entre as duas 
pessoas. O quadrado todo representa o conjunto de todos os pares possíveis 
(tudo em minuto, claro). Assim, como os pares são equiprováveis...


Basta calcular Área de S / Área do Quadrado = 11/36 para achar a resposta 
do problema.


Eu também fiz usando integral, mas ficou bem mais feio, tendo que dividir em 
casos.


Abraços,


Pedro Lazéra Cardoso


Conheça os novos produtos Windows Live. Clique aqui!


-- 
Fernando Gama


_
Deixe suas conversas mais divertidas. Baixe agora mesmo novos emoticons. É 
grátis!
http://specials.br.msn.com/ilovemessenger/pacotes.aspx

RE: [obm-l] Problema Bonito - Probabilidade

2009-05-19 Por tôpico Pedro Cardoso

Olá. Eu acho que é assim:
Problema: 
luiz silva escreveu:Duas pessoas marcam um encontro em um determinado local. 
Combinam que ambos deverão chgegar a este local entre 12 e13 h. Porém, qdo o 
1o. chegar ao local, irá esperar 10 min pelo outro. Caso o outro não chegeu ao 
local nete intervalo de tempo (10 min), o primeiro a chegar vai embora, e eles 
não conseguem se encontrar. Qual a probabilidade do encontro ocorrer ?
Bom, seja (a,b) o par que representa os dois instantes em que as duas pessoas 
chegaram, onde 0 = a,b = 60.Como a chance de ocorrência dos pares é igual, 
vale desenhar um quadrado de lado 60 no plano cartesiano,cujos vértices ficam 
nos pontos (0,0), (60,0), (60,60), (0,60), e fazer o seguinte:
Desenhar as retas y = x+10 e y = x-10.
A região do quadrado entre as retas (região S) forma o conjunto de pares (a,b) 
tal que abs(a-b) = 10, isto é, os paresque representam tempos de chegada para 
os quais há encontro entre as duas pessoas. O quadrado todo representa o 
conjunto de todos os pares possíveis (tudo em minuto, claro). Assim, como os 
pares são equiprováveis...
Basta calcular Área de S / Área do Quadrado = 11/36 para achar a resposta 
do problema.
Eu também fiz usando integral, mas ficou bem mais feio, tendo que dividir em 
casos.
Abraços,
Pedro Lazéra Cardoso
_
Descubra todas as novidades do novo Internet Explorer 8
http://brasil.microsoft.com.br/IE8/mergulhe/?utm_source=MSN%3BHotmailutm_medium=Taglineutm_campaign=IE8

RE: [obm-l] Combinatoria Pre-IME

2009-04-13 Por tôpico Pedro Cardoso


Olá, silas.

 

São 3 possibilidades:

 

moças separadas[i], duas (e não mais do que duas) moças juntas[ii], tres moças 
juntas[iii].

 

Sabemos que :

1) o total = [i] + [ii] + [iii] = 7!
2) [i] = 1440.

 

Ora, [iii] é mole.

Junte as moças e as trate como um 'elemento' só;
agora são 5 elementos, totalizando 5! arranjos possíveis;
permute as moças entre si: 3! (isto é, para cada arranjo acima, existem 3! = 6 
maneiras de as moças se sentarem)

 

3) [iii] = 3! * 5!

 

De '1)', '2)' e '3)', [ii] = total - [i] - [iii] = (7*6)*5! - (12)*5! - (6)*5! 
= 24*120 = 2880

 

(o problema é que, usando essa solução, se o cara - ou a moça - erra a 
primeira, erra também a segunda)

 

Abraços, 

 

Pedro Lazéra Cardoso

 


Date: Mon, 13 Apr 2009 22:33:40 -0300
Subject: Re: [obm-l] Combinatoria Pre-IME
From: silasgr...@gmail.com
To: obm-l@mat.puc-rio.br

Magistral, Benedito! Muito obrigado!


Bem, se não for muito inconveniente, vou postar a pergunta (b), que também está 
me dando uma surra:



Três moças e quatro rapazes estão num teatro e desejam, sentar-se, os sete, 
lado a lado, na mesma fila. Determine o número de maneiras pelas quais eles 
podem distribuir-se nos assentos de modo que:
  a) ...
  b) duas moças (e não mais do que duas moças) estejam sentadas juntas.
Resp.: 2880


2009/4/13 Benedito b...@ccet.ufrn.br



Silas,
 
Primeiro veja de quantos modos os rapazes podem sentar: 4! = 24.
Chame os rapazes de R1, R2, R3, R4, 
Agora, cada  moça só pode sentar entre os rapazes (ou à esquerda do primeiro ou 
à direita do último). 
---R1 --- R2 --- R3 --- R4  
Deste modo, há 5 lugares para a primeira moça sentar. Uma vez ocupada esta 
posição, restam 4 possíveis lugares para a segunda ocupar. Uma vez sentada a 
segunda moça, resta 3 posições (lugares)  nos quais a última moça pode ocupar. 
Assim, o total de possibilidades é 4! x 5 x 4 x 3 = 24 x 60 = 1440.
Benedito




- Original Message - 
From: Silas Gruta 
To: obm-l@mat.puc-rio.br 
Sent: Monday, April 13, 2009 3:48 PM
Subject: [obm-l] Combinatoria Pre-IME


Boa tarde a todos,
 
Tenho um aluno, cujo sonho é se formar pelo IME, extraordinariamente aplicado, 
uma verdadeira raridade numa escola pública! Faço o que posso para ajudá-lo, 
embora preparar alunos para o IME não seja, nem de perto, a minha 
especialidade. Bem, ele me apresentou um problema retirado de uma apostila de 
um curso Pré-IME/ITA de São Paulo, mas confesso que não estou conseguindo 
resolvê-lo mesmo depois de 13 dias de tetativas infrutíferas! Agradeço se 
puderem dar uma ajuda:
 
Três moças e quatro rapazes estão num teatro e desejam, sentar-se, os sete, 
lado a lado, na mesma fila. Determine o número de maneiras pelas quais eles 
podem distribuir-se nos assentos de modo que:
  a) duas moças nunca fiquem sentadas juntas; RESPOSTA: 1440
  b) ...
A pergunta (b) também é bem difícil, mas, se for o caso, apresento outro dia.
 
Obrigado!
-- 
Silas Gruta 

-- 
Silas Gruta

_
Novo Windows Live: Messenger 2009 e muito mais. Descubra!
http://www.windowslive.com.br

RE: [obm-l] Probabilidade - Joao e Maria

2009-03-19 Por tôpico Pedro Cardoso

Oi, Filipe. Eu saí fazendo coisas que nunca fiz, então talvez esteja errado.

 

Vamos torcer pra que não esteja e combinar umas notações:

 

A é o evento João vence

B é o evento João tira 6 na segunda rodada

P(evento) é a chance de 'evento' acontecer

(A inter B) é a intersecção dos eventos 'A' e 'B' 
 

Esse é um problema de probabilidade condicional - devemos calcular a chance de 
B acontecer, dado que A aconteceu.

Queromos, em notação matemática, P(B|A).

 

Por definição, P(B|A) = P(A inter B) / P(A)[i]

 

Dá um nó na cabeça ficar pensando na chance de João ter tirado 6 na segunda 
rodada, dado que ele venceu. O contrário, a chance de João ter vencido dado que 
ele tirou 6 na segunda rodada, é bem mais fácil (nesse caso, é banal, porque 
tirar 6 na segunda rodada é simplesmente vencer). Assim, 

 

P(A|B) = 1.

Mas P(A|B) = P(A inter B)/P(B)[ii]

 

Fazendo [i]/[ii], e rearrumando as coisas, 

 

P(B|A) = P(A|B)*P(A) / P(B)

 

P(A|B) = 1 (afinal, se o João tira 6 na segunda rodada, então ele vence o jogo)

P(B) = P(maria não tira 6)*P(João não tira 6)*P(Maria não tira 6)*P(João tira 
6) = 5^3/6^4

 

Agora vem P(A).

Bem, seja C a chance que a Maria tem de vencer.

Se ela não vence na primeira rodada, agora é o João que tem chance C de vencer.

 

P(joão vence)  = 5/6 * C

P(joão vence) + P(Maria vence) = C*5/6 + C = 1 .:. C = 6/11.

P(A) = 6/11

 

Assim, finalmente,  P(B|A) = P(A|B) P(A) / P(B) = (1)*(5^3/6^4) / (5/11) = 
25*11/6^4.

 

Isso dá aproximadamente 21,2%. Achei meio alto, Filipe!

 

Abraços,

 

Pedro Lazéra Cardoso

 

 

 


From: filipejunque...@msn.com
To: obm-l@mat.puc-rio.br
Subject: [obm-l] Probabilidade - Joao e Maria
Date: Wed, 18 Mar 2009 21:54:16 -0300





Eis o aqui o jogo.
 
Joao e Maria jogam um jogo de dados. Ganha quem tirar um “6” primeiro.
Exemplo:
 
Maria joga: tira 4
Joao joga: tira 3
Conclusao ninguém ganha
Maria joga: tira 6 
Conclusao: Ganhou
 
Com um dado apenas. Sabendo que Maria comeca jogando o dado na primeira rodada 
e que João ganha o jogo. 
Qual a probabilidade de João ter tirado um 6 na segunda rodada!?
 
 
Obrigado pessoal!
 
Obs: Eu só sei que a resposta não é 5/36 !
 
 
 
_
Receba GRÁTIS as mensagens do Messenger no seu celular quando você estiver 
offline. Conheça  o MSN Mobile!
http://mobile.live.com/signup/signup2.aspx?lc=pt-br

RE: [obm-l] Probabilidade - Joao e Maria

2009-03-19 Por tôpico Pedro Cardoso

Oi, Ralph.

 

Eu acho que uma rodada consiste em Maria e João jogarem.

Logo, Pr(J2) = Pr({xx x6}) = 5/6^4.

 

No resto, nossas respostas estão iguais.

 

Abraços,

 

Pedro.
 
 Date: Thu, 19 Mar 2009 19:36:41 -0300
 Subject: Re: [obm-l] Probabilidade - Joao e Maria
 From: ralp...@gmail.com
 To: obm-l@mat.puc-rio.br
 
 Olá a todos.
 
 Notação: x significa um número diferente de 6; 6 significa 6
 mesmo. Vou denotar a seqüências de lances de Maria e João, na ordem.
 Assim, se eu escrevo xx xx xx x6, isto significa que Maria e João se
 alternaram 3 vezes lançando números que não são 6, então Maria lançou
 outro número diferente de 6 e, finalmente, João ganhou tirando 6.
 
 Uma maneira de descrever o espaço amostral deste jogo é:
 
 U={6,x6,xx6,xxx6,6,...}
 
 cujas probabilidades são, respectivamente, 1/6, 5/6.1/6=5/36,
 5/6.5/6.1/6=25/216, e assim por diante (estou supondo dado justo e
 lançamentos independentes, que é a hipótese mais razoável já que
 ninguém disse nada a este respeito; note que estes números formam uma
 PG de razão 5/6 e soma 1 -- a soma ser 1 é um bom sinal!).
 
 Seja J o evento João ganhou e J2 o evento João ganhou na 2a rodada
 (de fato, note que J2={x6}). A pergunta é uma probabilidade
 condicional: quanto vale Pr(J2|J)?
 
 Bom, Pr(J2|J)=Pr(J2 e J)/Pr(J)=Pr(J2)/Pr(J) (pois J2 está contido em
 J, então J2 e J é o mesmo que J2).
 
 Agora Pr(J2)=Pr({x6})=5/36,
 enquanto Pr(J)=Pr(x2)+Pr(xxx2)+Pr(x2)+...=
 =5/36+5/36.25/36+5/36.25/36.25/36+...=5/36.1/(1-25/36) =5/11 (usei que
 isto é a soma dos termos de uma PG infinita, de razão 25/36).
 
 Assim, Pr(J2|J)=(5/36)/(5/11)=11/36. Esta é a resposta. Bom, eu acho
 -- vou deixar a galera ver se eu errei alguma bobagem no meio do
 caminho.
 
 Abraço,
 Ralph
 
 2009/3/18 Filipe Junqueira filipejunque...@msn.com:
  Eis o aqui o jogo.
 
 
 
  Joao e Maria jogam um jogo de dados. Ganha quem tirar um “6” primeiro.
 
  Exemplo:
 
 
 
  Maria joga: tira 4
 
  Joao joga: tira 3
 
  Conclusao ninguém ganha
 
  Maria joga: tira 6
 
  Conclusao: Ganhou
 
 
 
  Com um dado apenas. Sabendo que Maria comeca jogando o dado na primeira
  rodada e que João ganha o jogo.
 
  Qual a probabilidade de João ter tirado um 6 na segunda rodada!?
 
 
 
 
 
  Obrigado pessoal!
 
 
 
  Obs: Eu só sei que a resposta não é 5/36 !
 
 
 
 
 
 
 
 =
 Instruções para entrar na lista, sair da lista e usar a lista em
 http://www.mat.puc-rio.br/~obmlistas/obm-l.html
 =

_
Cansado de espaço para só 50 fotos? Conheça o Spaces, o site de relacionamentos 
com até 6,000 fotos!
http://www.amigosdomessenger.com.br

RE: [obm-l] probabilidade

2008-12-16 Por tôpico Pedro Cardoso

 
Oi,
 
jeffmaths. Eu fiz assim...
 
São n alunos votando em A, n votando em B.
Foram escolhidos aleatoriamente 4 deles, que chamaremos de a1,a2,a3,a4.
 
Vamos ver a chance de a1,a2 votarem em A; a3,a4 votarem em B.
Lembrando que a chance de alguém votar em Fulano é ''todo mundo que vota em 
Fulano'' / ''todo mundo''.
 
Ordem: a1, a2, a3, a4.
 
p = n/(2n) * (n-1)/(2n-1) * n/(2n-2) * (n-1)/(2n-3)
Mas também serve, por exemplo, (a2,a3) votando em A, (a1,a4) votando em B.
É fácil verificar que são 6 possibilidades no total (pode até fazer no braço!).
 
A resposta no final fica: 6*p = 6 * n/(2n) * (n-1)/(2n-1) * n/(2n-2) * 
(n-1)/(2n-3).
 
Para n=2 (quatro alunos), 6p = 1, o que faz sentido. 
Também faz sentido depender de n, porque, bem, imagina o IBGE entrevistando 4 
pessoas
na campanha para presidente...



Date: Mon, 15 Dec 2008 14:12:04 -0800From: jeffma...@yahoo.com.brsubject: 
[obm-l] probabilidadeTo: obm-l@mat.puc-rio.br




Ontem, recebi uma questão que ainda não resolvi, será que alguém pode me ajudar?
Lá vai: Um aluno entrevistou sua turma para saber a intenção de votos numa 
pesquisa para representante dela e notou que houve um empate técnico, metade da 
turma votaria no candidato A, enquanto que a outra metade votaria no candidato 
B. Bem, um instituto de pesquisa escolheu aleatoriamente 4 alunos dessa turma, 
qual será a probabilidade desse instituto chegar à mesma conclusão que o aluno?
Obrigado

Veja quais são os assuntos do momento no Yahoo! + Buscados: Top 10 - 
Celebridades - Música - Esportes
_
Cansado de espaço para só 50 fotos? Conheça o Spaces, o site de relacionamentos 
com até 6,000 fotos!
http://www.amigosdomessenger.com.br

RE: [obm-l] probabilidade

2008-12-16 Por tôpico Pedro Cardoso

Corrigindo: o IBGE não! O Ibope, o Datafolha etc...

From: pedrolaz...@hotmail.comto: ob...@mat.puc-rio.brsubject: RE: [obm-l] 
probabilidadeDate: Tue, 16 Dec 2008 13:57:35 -0200

 Oi, jeffmaths. Eu fiz assim... São n alunos votando em A, n votando em B.Foram 
escolhidos aleatoriamente 4 deles, que chamaremos de a1,a2,a3,a4. Vamos ver a 
chance de a1,a2 votarem em A; a3,a4 votarem em B.Lembrando que a chance de 
alguém votar em Fulano é ''todo mundo que vota em Fulano'' / ''todo mundo''. 
Ordem: a1, a2, a3, a4. p = n/(2n) * (n-1)/(2n-1) * n/(2n-2) * (n-1)/(2n-3)Mas 
também serve, por exemplo, (a2,a3) votando em A, (a1,a4) votando em B.É fácil 
verificar que são 6 possibilidades no total (pode até fazer no braço!). A 
resposta no final fica: 6*p = 6 * n/(2n) * (n-1)/(2n-1) * n/(2n-2) * 
(n-1)/(2n-3). Para n=2 (quatro alunos), 6p = 1, o que faz sentido. Também faz 
sentido depender de n, porque, bem, imagina o IBGE entrevistando 4 pessoasna 
campanha para presidente...

Date: Mon, 15 Dec 2008 14:12:04 -0800From: jeffma...@yahoo.com.brsubject: 
[obm-l] probabilidadeTo: obm-l@mat.puc-rio.br




Ontem, recebi uma questão que ainda não resolvi, será que alguém pode me ajudar?
Lá vai: Um aluno entrevistou sua turma para saber a intenção de votos numa 
pesquisa para representante dela e notou que houve um empate técnico, metade da 
turma votaria no candidato A, enquanto que a outra metade votaria no candidato 
B. Bem, um instituto de pesquisa escolheu aleatoriamente 4 alunos dessa turma, 
qual será a probabilidade desse instituto chegar à mesma conclusão que o aluno?
Obrigado

Veja quais são os assuntos do momento no Yahoo! + Buscados: Top 10 - 
Celebridades - Música - Esportes

Instale a Barra de Ferramentas com Desktop Search e ganhe EMOTICONS para o 
Messenger! É GRÁTIS!
_
Cansado de espaço para só 50 fotos? Conheça o Spaces, o site de relacionamentos 
com até 6,000 fotos!
http://www.amigosdomessenger.com.br

RE: [obm-l] Combinatoria e Prob

2008-10-16 Por tôpico Pedro Cardoso
Olá, 
 
Thaís, Lucas e amigos da lista.
Eu resolvi assim:
 
1)  (FUVEST-1997) Os trabalhos da diretoria de um clube são realizados por 
seis comissões. Cada diretor participa exatamente de duas comissões e cada duas 
comissões têm exatamente um diretor comum.
a) Quantos diretores tem o clube?
 
Seja n o número de participantes de uma comissão.
[1] Cada diretor participa exatamente de duas comissões.
[2] Cada duas comissões têm exatamente um diretor comum.
A priori, são 6*n diretores. Mas, por [1], contamos duas vezes cada um.
Logo, são 6*n/2 = 3*n diretores.
 
A priori, são 6*n diretores. Mas, por [2], para cada par de comissão, contamos 
duas vezes um mesmo diretor.
Logo, são 6*n - C(6,2) = 6n - 15 diretores.
 
Daí, 3*n = 6n - 15, donde n = 5 e são 15 os diretores.
 
b) Escolhendo-se, ao acaso, dois diretores, qual é a probabilidade de que eles 
sejam de uma mesma comissão?
 
 suponha que tenhamos um diretor já definido, então 2 comissões estão 
selecionadas. Agora vamos pegar as comissões do segundo diretor. As 
chances de pegarmos de primeira uma comissão selecionada é de 2/6 se pegarmos 
uma não selecionada de primeira (chances de 4/6), as chances de pegarmos uma 
selecionada é de 2/5: assim temos que as chances desse segundo caso acontecer 
são de 4*2/(5*6) = 4/15 Somando tudo: 2/6 + 4/15 = (10+8)/30=18/30=9/15=3/5
 
Hum... Lucas, eu penso que, como estamos sorteando diretores em vez de 
comissões, talvez sua resposta não esteja correta. Acho que você deveria 
considerar que um mesmo diretor não pode ser sorteado duas vezes. Além disso, 
você fez três sorteios, não?
 
Pela letra a), cada comissão tem 5 diretores e pelo enunciado [2] cada par de 
comissão tem apenas um único diretor em comum. Então, como o Lucas fez, vamos 
supor que o primeiro diretor, o José, foi sorteado. José participa de duas 
comissões e é o único diretor que trabalha nas duas ao mesmo tempo. Logo, 
existem 4 diretores em cada uma das duas comissões em que José está, todos eles 
distintos. 
 
Daí, são 8 diretores trabalhando com José e 6 não trabalhando com ele. Logo, a 
chance de pegarmos um diretor que não trabalha com José é 8/(6+8) = 4/7.
 
Eu supus que no sorteio o próprio José não pode ser sorteado.
 
Abraços,
 
Pedro.
_
Conheça o Windows Live Spaces, a rede de relacionamentos do Messenger!
http://www.amigosdomessenger.com.br/

[obm-l] Notação de Leibniz

2008-09-25 Por tôpico Pedro Cardoso
 
Olá.
 
Quero dizer primeiro que eu procurei nos meus dois livros de cálculo (James 
Stewart e o da PUC) e na internet em geral a definição de dy/dx (ou df/dg, 
tanto faz) e não achei algo que explicitasse isso com clareza o suficiente para 
mim. Me viro bem com os outros conceitos de derivação, usando f(x) e f´(x), mas 
a notação de Leibniz ainda me causa certa confusão.
 
[eu vou usar a notação lim[b-c]{f/g} sendo limite de (f/g) quando 'b' tende a 
'c']
 
Enfim, o Stewart diz que dy/dx, quando y está em função de x, é definido como...
 
dy/dx = lim[delta(x)-0]{delta(y)/delta(x)}
 
Peruntas:
 
[1] Eu posso interpretar dx como lim[delta(x)-0]{ delta(x) }
[2] Se sim, então posso dizer que dy/dx = lim[delta(y)-0]{ delta(y) } / 
lim[delta(x)-0]{ delta(x) }
[3] Se sim, por que não posso 'cortar' du em (dy/du)*(du/dx) ?
[4] Qual a definição precisa de dy/dx?
[5] Algo mais de interessante para acrescentar?
 
Abraços,
 
Pedro Lazéra Cardoso.
_
Receba GRÁTIS as mensagens do Messenger no seu celular quando você estiver 
offline. Conheça  o MSN Mobile!
http://mobile.live.com/signup/signup2.aspx?lc=pt-br

[obm-l] Álgebra Linear

2008-05-04 Por tôpico Pedro Cardoso
Saudações.
 
Vai aqui um de álgebra linear. Se possível, gostaria que a solução usasse 
poucos conceitos
avançados (quanto mais elementar, melhor!). 
 
Problema: 
 
Seja T:R^3-R^3 uma transformação linear. Provar que,
se a Im(T) não é o próprio R^3, então existe um vetor v, não nulo,
tal que T(v) = 0 (o próprio vetor nulo).
 
Im(T) significa imagem de T.
 
Obrigado,
 
Pedro Lazéra Cardoso
_
Conheça o Windows Live Spaces, a rede de relacionamentos do Messenger!
http://www.amigosdomessenger.com.br/

[obm-l] Algebra Linear e Desigualdade de Schwarz

2008-03-03 Por tôpico Pedro Cardoso
Caros amigos da lista,
 
saudações! Queria a ajuda de vocês em dois problemas, nos quais a minha dúvida 
consiste em saber com exatidão o que o enunciado exige de mim. Um é de álgebra 
linear, outro é de, bem, desigualdade de couchy-schwarz (que tópico da álgebra 
isso seria?).
 
1- Determine sistemas lineares de equações com duas incógnitas (x,y) cujas 
soluções sejam da forma (1,3) (solução única), (t,2t),(t,3). Comentário: na 
verdade, vendo a solução de uma dessas, acho que entenderia o que é para fazer 
nas outras. Até resolvi, mas a resposta não parecia estar bonita e soava óbvia 
demais.
 
2- Dê a interpretação geométrica da desigualdade de Couchy-Schwarz para n=2, 
n=3.
 
Abraços,
 
Pedro Lazéra cardoso.
 
_
Cansado de espaço para só 50 fotos? Conheça o Spaces, o site de relacionamentos 
com até 6,000 fotos!
http://www.amigosdomessenger.com.br

RE: [obm-l] Conjuntos (básico)

2008-01-21 Por tôpico Pedro Cardoso

Amigos da lista, preciso da ajuda de vocês na solução de um problema. Como o 
texto é longo, vou resumir, tomando cuidado para não modificar o enunciado. 
Numa pesquisa, existem três propostas. Uma pessoa pesquisada pode ser favorável 
a uma, duas ou todas as propostas, ou seja, as propostas não são mutuamente 
excludentes. No resumo, 

78% são favoráveis a pelo menos uma proposta
50% são favoráveis à proposta A
30% são favoráveis à proposta B

20% são favoráveis à proposta C
5% favorável a todas as propostas

Questão: quantos % são favoráveis a mais de uma proposta?

O problema é do concurso público realizado pela ESAF, 2003/2004. De acordo com 
o gabarito, a resposta é 5%, mas achei 17%.

Obrigado,
Pedro Lazéra Cardoso

_
Cansado de espaço para só 50 fotos? Conheça o Spaces, o site de relacionamentos 
com até 6,000 fotos!
http://www.amigosdomessenger.com.br

RE: [obm-l] Re: [obm-l] Combinatória

2007-12-17 Por tôpico Pedro Cardoso
 
 
Só pra agradecer ao Ralph e ao Rogiro cientista pelos explicações e pelos links 
(sobre o problema das partições de números naturais). Vou tentar entender um 
pouco mais o assunto dentro do que a matemática que domino permite. 
 
Obrigado aos dois.
_
Receba GRÁTIS as mensagens do Messenger no seu celular quando você estiver 
offline. Conheça  o MSN Mobile!
http://mobile.live.com/signup/signup2.aspx?lc=pt-br

RE: [obm-l] probabilidades

2007-12-05 Por tôpico Pedro Cardoso


Opa, Ralph.
 
Eu já tinha lido outros e-mails seus com essa advertência, mas só agora percebi 
que nesse caso* seu alerta também era válido. Sou um dos alunos novos que 
aprenderam assim - que probabilidade é caso favorável/caso possível. Vou até 
testar meu professor!
 
*me refiro ao e-mail do ralph sobre o problema de probabilidade enviado pelo 
crmoraes, sobre sorteio de bolinha de gude.
 
Enfim, muito obrigado (pelo elogio e pela aula).
_
Veja mapas e encontre as melhores rotas para fugir do trânsito com o Live 
Search Maps!
http://www.livemaps.com.br/index.aspx?tr=true

RE: [obm-l] probabilidade

2007-12-04 Por tôpico Pedro Cardoso
Olá...
 
De quantas maneiras eu posso escrever um número N como a soma de fatores, não 
importando a ordem deles?
 
Como a pergunta ficou mal formulada, eu dou exemplos:[2] = 1+1 = 2+0.
[3] = 1+1+1 = 1+2 = 3+0
[4] = 1+1+1+1 = 2+1+1 = 3+1 = 4+0 = 2+2
...
[N] = ???
_
Veja mapas e encontre as melhores rotas para fugir do trânsito com o Live 
Search Maps!
http://www.livemaps.com.br/index.aspx?tr=true

RE: [obm-l] probabilidade

2007-12-04 Por tôpico Pedro Cardoso
Cmoraes, eu recomendo que você escreve no google soluções inteiras 
não-negativas.
 
I) Depende do número de bolinhas. Se houver mais de 9 bolinhas de cada cor, 
tudo bem. Caso contrário, fica mais complicado, eu acho. Supondo que sejam mais 
de 9 de cada cor...
 
Sejam x1 o número de bolinhas verdes, x2  o número de amarelas, x3 o de azuis, 
x4 o de brancas. 
P(A) é a probabilidade de ocorrer o evento A. 
Além disso, o sinal '= significa 'maior ou igual'.
 
x1+x2+x3+x4 = 10 
 
O número de soluções inteiras não-negativas dessa equação corresponde ao número 
de casos possíveis para os grupos de 10 bolinhas (desde que a ordem das 
bolinhas não importe). Casos possíveis = 13!/(3!10!) = 260.
 
P(não haver quatro cores) = 1 - P(haver quatro cores)
 
Para que hajam quatro cores, devemos ter x1,x2,x3,x40. Considere y um inteiro 
maior ou igual a 0.
Assim, satisfazendo as condições do problema, x1 = y1+1; x2 = y2+1; x3 = y3+1; 
x4 = y4+1
 
Como x1+x2+x3+x4 = 10, (y1+1)+(y2+1)+(y3+1)+(y4+1) = 10 .:. y1+y2+y3+y4 = 6 
(y=0)
 
Os casos favoráveis são as soluções dessa última equação.
Número de casos favoráveis = 9!/(3!6!) = 84.
 
P(haver quatro cores) = 84/260 = 21/65
P(não haver quatro cores) = 1 - 21/65 = 44/65.
 
O II é parecido, então, entendendo o I, acho que você consegue resolvê-lo.
 
*Eu considerei x=10 porque, caso contrário, na equação x1+x2+x3+x4 =10, eu 
teria que trabalhar com vários casos.
A solução x1 =10, x2,x3,x4 = 0 não valeria, por exemplo. 
 
Até. Espero ter ajudado.
_
Conheça o Windows Live Spaces, a rede de relacionamentos conectada ao Messenger!
http://spaces.live.com/signup.aspx

[obm-l] [obm-l] Horário das Provas

2007-10-17 Por tôpico Pedro Cardoso

Olá,

queria esclarecer uma dúvida a respeito do horário de realização da terceira 
fase da obm. Segundo o site da OBM, quem for do nível 3 (não importa os 
outros níveis para este caso) realizará a prova no Sábado 27 e Domingo 28 
de outubro às 14 horas (horário de Brasília).


Acontece que quem for prestar vestibular para a PUC, inclusive querendo 
fazer matemática ou engenharia, terá provas dia 28, das 15H às 19H. Alguma 
pode ser feita ou o estudante, nesse caso, terá de optar por uma das provas?


Grato,

Pedro Lazéra Cardoso.

_
Descubra como mandar Torpedos do Messenger para o celular! 
http://mobile.msn.com/


=
Instruções para entrar na lista, sair da lista e usar a lista em
http://www.mat.puc-rio.br/~obmlistas/obm-l.html
=


[obm-l] [obm-l] OBM, fase 2, nível 3, última quest ão

2007-09-19 Por tôpico Pedro Cardoso

Olá.

Eu tenho uma dúvida em relação à última questão do nível 3 da segunda fase 
da obm:


O enunciado:

Em um certo país há 21 cidades e o governo pretende construir n estradas 
(todas de mão dupla),
sendo que cada estrada liga exatamente duas das cidades do país. Qual o 
menor valor de n para que,
independente de como as estradas sejam construídas, seja possível viajar 
entre quaisquer duas

cidades (passando, possivelmente, por cidades intermediárias)?

Ora, por que ligar uma cidade a todas as outras vinte, construindo assim 20 
estradas, não satisfaz todas as condições do problema? Seria possível viajar 
entre quaisquer duas cidades, desde que fosse possível passar pela cidade à 
qual todas as outras estão ligadas (o que o enunciado permite fazer).


Pedro Lazéra Cardoso

_
Descubra como mandar Torpedos do Messenger para o celular! 
http://mobile.msn.com/


=
Instruções para entrar na lista, sair da lista e usar a lista em
http://www.mat.puc-rio.br/~nicolau/olimp/obm-l.html
=


[obm-l] [obm-l] Gabarito da obm - nível 2

2007-09-18 Por tôpico Pedro Cardoso

Olá.

E o gabarito da prova, não iria sair hoje, terça? O pessoal da lista que 
está ligado de alguma forma à organização da olimpíada teria alguma 
informação para fornecer aos interessados nas provas de segunda fase?


Agradeço,

Pedro Lazéra Cardoso.

_
Mande torpedos SMS do seu messenger para o celular dos seus amigos 
http://mobile.msn.com/


=
Instruções para entrar na lista, sair da lista e usar a lista em
http://www.mat.puc-rio.br/~nicolau/olimp/obm-l.html
=


[obm-l] RE: [obm-l] [obm-l] Gabarito da obm - níve l 2

2007-09-18 Por tôpico Pedro Cardoso
Só corrigindo o que já deve ter ficado claro: é gabarito da fase 2., não do 
nível 2.


Abraços,

Pedro Lazéra Cardoso.



From: Pedro Cardoso [EMAIL PROTECTED]
Reply-To: obm-l@mat.puc-rio.br
To: obm-l@mat.puc-rio.br
Subject: [obm-l] [obm-l] Gabarito da obm - nível 2
Date: Tue, 18 Sep 2007 19:54:58 -0300

Olá.

E o gabarito da prova, não iria sair hoje, terça? O pessoal da lista que 
está ligado de alguma forma à organização da olimpíada teria alguma 
informação para fornecer aos interessados nas provas de segunda fase?


Agradeço,

Pedro Lazéra Cardoso.

_
Mande torpedos SMS do seu messenger para o celular dos seus amigos 
http://mobile.msn.com/


=
Instruções para entrar na lista, sair da lista e usar a lista em
http://www.mat.puc-rio.br/~nicolau/olimp/obm-l.html
=


_
Seja um dos primeiros a testar o novo Windows Live Mail Beta- grátis. Acesse 
http://www.ideas.live.com/programpage.aspx?versionId=5d21c51a-b161-4314-9b0e-4911fb2b2e6d


=
Instruções para entrar na lista, sair da lista e usar a lista em
http://www.mat.puc-rio.br/~nicolau/olimp/obm-l.html
=


Re: [obm-l] Cubo de Rubik

2007-09-13 Por tôpico Pedro Cardoso

Douglas,

valeu pela ajuda, de novo. Acho que também entendi parcialmente o caminho da 
solução. Os oito cubos de quina podem ser permutados entre si, gerando 8! 
possibilidades. Os 12 cubos de arestas podem ser permutados entre si, 
gerando 12! possibilidades. Além disso, você pode 'girar' esses cubos, o que 
gera mais 3^8 possibilidades para os cubos de quina (pois são 3 cores) e 
2^12 possibilidades para os cubos de aresta (pois são 2 cores). Só que, para 
mim, isso geraria alguns casos repetidos.


Continuo, portanto, apenas com uma idéia vaga da solução.


From: Douglas Ribeiro Silva [EMAIL PROTECTED]
Reply-To: obm-l@mat.puc-rio.br
To: obm-l@mat.puc-rio.br
Subject: Re: [obm-l] Cubo de Rubik
Date: Thu, 13 Sep 2007 01:14:57 -0300

Olá Pedro!

Eu tenho uma idéia de onde vieram alguns números mas não entendo
perfeitamente a formula como um todo... vamos lá:

8!12!(3^8)(2^12)/2*3*2

Os numeros do numerador surgem do fato que o cubo possui 27 cubos
menores. O cubo central não conta para as possiveis permutacoes, logo
temos 26. Note também que o cubo que representa o centro de cada face
é fixo, por isso temos 6 cubos que não irão participar da contagem.
Logo resta 20 cubos.
Desses 20, 8 são cubos de quina, que possuem 3 cores e 12 são cubos de
aresta que possuem duas cores tenho certeza que é daí que surgem o 8!
12! 3^8 e 2^12

O restante eu não entendi muito bem o que foi feito e gostaria que
alguém explicasse melhor também.

Abraços!



Em 12/09/07, Pedro Cardoso[EMAIL PROTECTED] escreveu:
 Douglas,

 muito obrigado pelo link, mas parece que o site simplesmente diz quantas 
são
 as possibilidades, sem mostrar como o resultado foi obtido. Então, se 
alguém

 da lista puder expor uma explicação para se chegar ao número correto de
 arranjos de um cubo mágico, ainda agradeço.

 Pedro Lazéra Cardoso

 From: Douglas Ribeiro Silva [EMAIL PROTECTED]
 Reply-To: obm-l@mat.puc-rio.br
 To: obm-l@mat.puc-rio.br
 Subject: Re: [obm-l] Cubo de Rubik
 Date: Tue, 11 Sep 2007 03:19:58 -0300
 
 Ola Pedro!
 
 Sugiro que visite o link http://mathworld.wolfram.com/RubiksCube.html
 que tem varias informações sobre o cubo incluindo o numero de
 combinações possiveis.
 
 Abraços!
 
 Em 11/09/07, Pedro Cardoso[EMAIL PROTECTED] escreveu:
   Olá.
  
   Peço desculpas se alguém já apresentou à lista essa questão - não
 consigo
   achar muita coisa naquele site indicado para pesquisar assuntos
   anteriormente discutidos por aqui. De qualquer forma, aí vai...
  
   Quantas são as possibilidades de arranjo de um cubo mágico, ou cubo 
de
   rubik? Lembrando que um cubo mágico é um cubo cujas faces são 
divididas

 em 9
   quadrados iguais, pintados com uma única cor, entre 6 disponíveis. 
Cada

 cor
   deve ser usada um número idêntico de vezes (9 vezes).
  
   Acredito que seja dispensável, mas, se alguém quiser ver uma imagem:
   http://tatooblue.blogs.sapo.pt/arquivo/cubo_magico.jpg
  
   Queria saber se minha solução esta certa (acho que não).
   Primeiramente, fiz 54!. Depois, dividi por 9! seis vezes, para não
 contar os
   casos em que quadrados de mesma cor foram trocados de posição.
 Finalmente,
   dividi ainda por 24, que é o número de vezes que posso obter
 configurações
   'diferentes' por rotação. Achei um número grande demais - cerca de
 10^38, de
   acordo com a calculadora do windows.
  
   Pedro Lazéra Cardoso

 _
 Inscreva-se no novo Windows Live Mail beta e seja um dos primeiros a 
testar

 as novidades-grátis. Saiba mais:
 
http://www.ideas.live.com/programpage.aspx?versionId=5d21c51a-b161-4314-9b0e-4911fb2b2e6d


 
=

 Instruções para entrar na lista, sair da lista e usar a lista em
 http://www.mat.puc-rio.br/~nicolau/olimp/obm-l.html
 
=



=
Instruções para entrar na lista, sair da lista e usar a lista em
http://www.mat.puc-rio.br/~nicolau/olimp/obm-l.html
=


_
Verificador de Segurança do Windows Live OneCare: verifique já a segurança 
do seu PC! http://onecare.live.com/site/pt-br/default.htm


=
Instruções para entrar na lista, sair da lista e usar a lista em
http://www.mat.puc-rio.br/~nicolau/olimp/obm-l.html
=


Re: [obm-l] Cubo de Rubik

2007-09-12 Por tôpico Pedro Cardoso

Douglas,

muito obrigado pelo link, mas parece que o site simplesmente diz quantas são 
as possibilidades, sem mostrar como o resultado foi obtido. Então, se alguém 
da lista puder expor uma explicação para se chegar ao número correto de 
arranjos de um cubo mágico, ainda agradeço.


Pedro Lazéra Cardoso


From: Douglas Ribeiro Silva [EMAIL PROTECTED]
Reply-To: obm-l@mat.puc-rio.br
To: obm-l@mat.puc-rio.br
Subject: Re: [obm-l] Cubo de Rubik
Date: Tue, 11 Sep 2007 03:19:58 -0300

Ola Pedro!

Sugiro que visite o link http://mathworld.wolfram.com/RubiksCube.html
que tem varias informações sobre o cubo incluindo o numero de
combinações possiveis.

Abraços!

Em 11/09/07, Pedro Cardoso[EMAIL PROTECTED] escreveu:
 Olá.

 Peço desculpas se alguém já apresentou à lista essa questão - não 
consigo

 achar muita coisa naquele site indicado para pesquisar assuntos
 anteriormente discutidos por aqui. De qualquer forma, aí vai...

 Quantas são as possibilidades de arranjo de um cubo mágico, ou cubo de
 rubik? Lembrando que um cubo mágico é um cubo cujas faces são divididas 
em 9
 quadrados iguais, pintados com uma única cor, entre 6 disponíveis. Cada 
cor

 deve ser usada um número idêntico de vezes (9 vezes).

 Acredito que seja dispensável, mas, se alguém quiser ver uma imagem:
 http://tatooblue.blogs.sapo.pt/arquivo/cubo_magico.jpg

 Queria saber se minha solução esta certa (acho que não).
 Primeiramente, fiz 54!. Depois, dividi por 9! seis vezes, para não 
contar os
 casos em que quadrados de mesma cor foram trocados de posição. 
Finalmente,
 dividi ainda por 24, que é o número de vezes que posso obter 
configurações
 'diferentes' por rotação. Achei um número grande demais - cerca de 
10^38, de

 acordo com a calculadora do windows.

 Pedro Lazéra Cardoso


_
Inscreva-se no novo Windows Live Mail beta e seja um dos primeiros a testar 
as novidades-grátis. Saiba mais: 
http://www.ideas.live.com/programpage.aspx?versionId=5d21c51a-b161-4314-9b0e-4911fb2b2e6d


=
Instruções para entrar na lista, sair da lista e usar a lista em
http://www.mat.puc-rio.br/~nicolau/olimp/obm-l.html
=


[obm-l] Cubo de Rubik

2007-09-10 Por tôpico Pedro Cardoso

Olá.

Peço desculpas se alguém já apresentou à lista essa questão - não consigo 
achar muita coisa naquele site indicado para pesquisar assuntos 
anteriormente discutidos por aqui. De qualquer forma, aí vai...


Quantas são as possibilidades de arranjo de um cubo mágico, ou cubo de 
rubik? Lembrando que um cubo mágico é um cubo cujas faces são divididas em 9 
quadrados iguais, pintados com uma única cor, entre 6 disponíveis. Cada cor 
deve ser usada um número idêntico de vezes (9 vezes).


Acredito que seja dispensável, mas, se alguém quiser ver uma imagem:
http://tatooblue.blogs.sapo.pt/arquivo/cubo_magico.jpg

Queria saber se minha solução esta certa (acho que não).
Primeiramente, fiz 54!. Depois, dividi por 9! seis vezes, para não contar os 
casos em que quadrados de mesma cor foram trocados de posição. Finalmente, 
dividi ainda por 24, que é o número de vezes que posso obter configurações 
'diferentes' por rotação. Achei um número grande demais - cerca de 10^38, de 
acordo com a calculadora do windows.


Pedro Lazéra Cardoso

_
Verifique já a segurança do seu PC com o Verificador de Segurança do Windows 
Live OneCare! http://onecare.live.com/site/pt-br/default.htm


=
Instruções para entrar na lista, sair da lista e usar a lista em
http://www.mat.puc-rio.br/~nicolau/olimp/obm-l.html
=


Re: [obm-l] ALUNOS

2007-08-22 Por tôpico Pedro Cardoso

Saulo, nessa questão eu acho que você deve enxergar duas coisas:

1- existe uma ordem coerente para colorir as quatro regiões do mapa;
2- é aconselhável dividir o problema em dois casos.

Vou supor que esse mapa é o círulo trigonométirco, só pra gente já saber 
localizar cada região (são os 4 quadrantes). Pintar, nesta ordem, os 
quadrantes I,II,III,IV não é inteligente: quando eu for pintar o IV, não vai 
ser possível dizer quantas são as possibilidades, já que não sabemos se I e 
III foram pintados com a mesma cor ou com cores diferentes. Divido então em 
dois casos:


Quadrantes I e III de cor diferente (caso 1); quadrantes I e III de cor 
igual (caso 2).


Caso 1: I (T cores); III (T-1 cores); II (T-2 cores); IV (T-2 cores) = 
t(t-1)(t-2)(t-2)
Caso 2: I (T cores); III (1 cor); II (T-1 cores); IV (T-1 cores) = 
t(t-1)(t-1)


Caso 1 + Caso 2 = t(t-1)[(t-2)(t-2) + (t-1)] = t(t-1)(t^2-3t+3)

a) Podemos pintar o mapa de t(t-1)(t^2-3t+3), se eu não errei conta
b) O menor valor de t é 2, mas isso você pode fazer no braço, usando duas 
cores pra pintar o mapa. Não dá nem pra pintar errado.



Problema:

Ah outra dúvida minha é sobre uma questão do livro Análise
Combinatória e Probabilidade da coleção do professor de matemática
do saudoso Morgado e outros grandes professores.
É a questão 27 do capítulo 2 que é assim:

A figura 2.3 mostra um mapa com 4 países ( é um círculo dividido em
4 partes iguais)
a) De quantos modos esse mapa pode ser colorido (cada país com uma
cor, países com uma linha fronteira comum não podem ter a mesma cor)
se dispomos de T cores diferentes?
b) Qual o menor valor de T que permite colorir o mapa?

Bem achei a resposta da letra a diferente do gabarito talvez esteja
errado minha resolução mas gostaria de saber se alguém aqui já fez
esta questão e achou igual a do gabarito.



Abraços,

Pedro Lazéra Cardoso

_
Seja um dos primeiros a testar o novo Windows Live Mail Beta- grátis. Acesse 
http://www.ideas.live.com/programpage.aspx?versionId=5d21c51a-b161-4314-9b0e-4911fb2b2e6d


=
Instruções para entrar na lista, sair da lista e usar a lista em
http://www.mat.puc-rio.br/~nicolau/olimp/obm-l.html
=


RE: [obm-l] ALUNOS

2007-08-21 Por tôpico Pedro Cardoso

Olá, arkon. Achei duas soluções...


Solução 1 (roubada):


Como a nota dos alunos parece ser dada até a primeira casa decimal, a soma 
das novas notas dos alunos reprovados tem de ser dada também até a primeira 
casa decimal. Achamos a soma multiplicando a média pelo número de alunos. 
Dessa forma, 6,88*a (a é o número de alunos reprovados) tem de ir apenas até 
a segunda casa decimal, com 'a' variando entre 0 e 8, excluindo os extremos 
(o ou 8). Nossa incógnita (a) só pode valer 5.


Logo, são 5 alunos reprovados e 15 aprovados.


Solução 2 (trabalhosa, porém mais organizada)


x_k detona a nota do aluno k. São 20 alunos, dos quais, após a mudança das 
notas, n migraram do grupo dos reprovados para o grupo dos aprovados. 
Calculamos a nova média somando as notas dos X alunos que seriam aprovados 
com a mudança, dividindo o resultado por X e somando a ele 0,5. Entendido 
isso, podemos escrever:


A segunda média dos reprovados = 6,88 = (x_1+...+x_k)/(8-n) + 0,5
A segunda média dos aprovados = 8 = (x_(k+1)+...+x_20)/(12+n) + 0,5

Note que x_k representa a nota antiga do aluno k. Estou somando o 0,5 depois 
e incluindo o aluno antigo cuja nota era anteriormente menor do que 7, mas 
suficientemente grande para ser aprovado após a mudança, no grupo dos novos 
aprovados.


Logo,

[I] (6,88-0,5)*(8-n) = x_1+...+x_k
[II] (8-0,5)*(12+n) = x_(k+1)+...+x_20

Somando, I+II = (6,88-0,5)*(8-n) + (8-0,5)*(12+n) = [x_1+...+x_k] + 
[x_(k+1)+...+x_20]

Então, (6,88-0,5)*(8-n) + (8-0,5)*(12+n) = soma das notas antigas.

Em relação às notas antigas, eram 8 alunos com média 6,5 e 12 alunos com 
média 7,7. Teremos:
(6,88-0,5)*(8-n) + (8-0,5)*(12+n) = 8*6,5 + 12*7,7. Fazendo um bando de 
contas...

n(7,5-6,38) + 8*6,38 + 12*7,5 = 8*6,5 + 12*7,7 .:. n(1,12) = 3,36.

Olha que emocionante... deu certinho. n = 3,36/1,12 = 3. No fim, 15 alunios 
foram aprovados.


Problema:

Numa sala composta por 20 alunos, 8 alunos foram reprovados pois a nota 
mínima para ser aprovado é 7. A média dos reprovados foi 6,5 enquanto a dos 
aprovados foi 7,7. O professor considerou que uma questão foi mal formulada 
e, portanto, acrescentou 0,5 na nota de todos os alunos. Assim, a média dos 
aprovados e dos reprovados passou a ser 8 e 6,88, respectivamente. Calcular 
o número de alunos que conseguiram se aprovar após o acréscimo de 0,5 em 
suas notas.


Abraços,

Pedro Lazéra Cardoso

_
Verificador de Segurança do Windows Live OneCare: verifique já a segurança 
do seu PC! http://onecare.live.com/site/pt-br/default.htm


=
Instruções para entrar na lista, sair da lista e usar a lista em
http://www.mat.puc-rio.br/~nicolau/olimp/obm-l.html
=


[obm-l] Re: [obm-l] Re: [obm-l] Combinatória e Pro babilidade

2007-08-21 Por tôpico Pedro Cardoso
Nehab, muito obrigado. Parece ser mesmo isso que você falou - é que eu nunca 
tinha estudado probabilidade binominal antes.


Vou me informar.

Grato,

Pedro Lazéra Cardoso

_
Descubra como mandar Torpedos SMS do seu Messenger para o celular dos seus 
amigos. http://mobile.msn.com/


=
Instruções para entrar na lista, sair da lista e usar a lista em
http://www.mat.puc-rio.br/~nicolau/olimp/obm-l.html
=


[obm-l] Re: [obm-l] Combinatória e Probabilidade

2007-08-17 Por tôpico Pedro Cardoso
Salhab, primeiro obrigado por tentar resolver o problema. Segundo, vou 
procurar te mostrar até onde cheguei, para ver se você consegue, porque 
conhece muito mais do que eu, solucionar de vez a questão.


A chance de se acertar n questões - P(n) - é igual a (1/4)^n * (3/4)^(60-n) 
* C(60,n). Esse fator
C(60,n) entra porque não foi estabelecida nenhuma ordem de acerto. 
Reescrevendo, separando o que varia do que é constante, temos:


P(n) = (1/4)^n * (3/4)^(60-n) * C(60,n) = (1* 3^60 * 60!) / [4^60 * 3^n * n! 
* (60-n)!]
Veja que, dessa forma, o numerador é constante e somente uma parte do 
denominador é variável.


P(n) = (1* 3^60 * 60!) / [4^60 * f(n)], onde f(n) = 3^n * n! * (60-n)!

O problema passa a ser minimizar f(n), n variando de 0 a 60. Para a+b = 60, 
ab, C(60,a) = C(60,b), mas 3^a  3^b. Fica bem óbvio, então (embora isso já 
fosse algo intuitivo), que só temos de testar os valores até n = 30. Para n 
= 31, por exemplo, f(29)  f(31) = P(29)P(31).


Sobre intuitivmente acertarmos 1 questão a cada quatro... Vamos supor uma 
prova composta de 4 questões, cada uma com quatro alternativas. Nesse caso, 
f(n) = 3^n * n! * (4-n)!, e só precisamos testar até n = 2.


Testando n=1... f(1) = (3 * 1! * 3!) = 18
Testando n=2... f(2) = (3^2 * 2! * 2!) = 36.

De fato, acertar uma questão é o mais provável. Acertar 15 de 60 também 
seria portanto o resultado mais provável para a UERJ. Acho, aliás, que eu 
poderia supor ser essa prova de 60 questões a junção de 15 provas de 4 
questões. E, testando alguns valores, lembrando que f(n) tem de ser 
minimizado, temos:


f(14) = 3^14 * 14! * 46!
f(15) = 3^15 * 15! * 45! = f(14)*45/46
f(16) = 3^16 * 16! * 44! = f(14)*48/46

f(15)f(14)f(16), o que faz sentido. A chance deve crescer de 1 até 15 e 
descrescer de 15 até 60.


Mas eu ainda queria saber como minimizar f(n) = 3^n * n! * (60-n)!

Grato,

Pedro Lazéra Cardoso

_
Mande torpedos SMS do seu messenger para o celular dos seus amigos 
http://mobile.msn.com/


=
Instruções para entrar na lista, sair da lista e usar a lista em
http://www.mat.puc-rio.br/~nicolau/olimp/obm-l.html
=


[obm-l] =

2007-08-16 Por tôpico Pedro Cardoso
Amigos da lista, eis aqui outra dúvida minha. Para o pessoal que gosta de 
saber as origens do problema, esse veio da minha cabeça mesmo, um dia após 
fazer uma prova de vestibular.


Vamos tomar como exemplo a prova da UERJ (Universidade Estadual do Rio de 
Janeiro). Ela é composta por 60 questões de múltipla escolha, cada uma com 4 
opções, das quais apenas uma é correta.


Pergunta: qual é o resultado mais provável da prova para um candidato que 
tenha marcado ao acaso apenas uma alternativa em cada uma das 60 questões? 
Enfim, quem chutou tudo acertou provavelmente quantas questões?


Grato,

Pedro Lazéra Cardoso

_
Verifique já a segurança do seu PC com o Verificador de Segurança do Windows 
Live OneCare! http://onecare.live.com/site/pt-br/default.htm


=
Instruções para entrar na lista, sair da lista e usar a lista em
http://www.mat.puc-rio.br/~nicolau/olimp/obm-l.html
=


[obm-l] [obm-l] OBM 1999 Fase 2, Nível 3, Problema 5

2007-08-08 Por tôpico Pedro Cardoso

Olá,

estou com dúvidas em relação à solução da banca para o problema 5 da segunda 
fase do nível 3, na OBM de 1999. Vou indicar a parte que não entendi abaixo 
e deixar no final da mensagem a versão integral da questão.


Notação: m_n é o n-ésimo termo de uma sequência.

A banca conclui, com os dados do problema, que m_(n+1) = [1 - m_n]/2. Até aí 
tudo bem. Disso, ela chega em...


... m_n = [1 - (-2)^(2-n)] / 3, sem explicar como. Foi exatamente essa 
passagem que não compreendi. Agradeceria a quem me explicasse como se chegou 
a isso. Abaixo segue o enunciado:


José tem três pares de óculos, um magenta, um amarelo e um ciano. Todo dia 
de manhã ele escolhe um ao acaso, tendo apenas o cuidado de nunca usar o 
mesmo que usou no dia anterior. Se dia primeiro de agosto ele usou o 
magenta, qual a probabilidade de que dia 31 de agosto ele volte a usar o 
magenta?


Pedro Lazéra Cardoso

_
Verificador de Segurança do Windows Live OneCare: combata já vírus e outras 
ameaças! http://onecare.live.com/site/pt-br/default.htm


=
Instruções para entrar na lista, sair da lista e usar a lista em
http://www.mat.puc-rio.br/~nicolau/olimp/obm-l.html
=


[obm-l] RE: [obm-l] ajuda (polinômio)

2007-06-14 Por tôpico Pedro Cardoso

Kleber, nesse problema você pode usar uma ferramente poderosa para fatorar:

num polinômio qualquer, se a = b zera esse polinômio, (a - b) é fator.

Em vez de testar valores aleatórios para x,y ou z, é melhor tentar usar 
alguma coisa mais genérica. Seja y = -x


P(x,y,z) = (x^5 + (-x)^5 + z^5) - (x + (-x) + z)^5 = z^5 - z^5 = 0.
Se y = -x zera o polinômio, (y+x) é fator.
Analogamente, (y+z) e (z+x) são fatores também.

P(x,y,z) = (x+y)(x+z)(z+y)(...)

Resposta: letra b)

Pedro Lazéra Cardoso

_
O Windows Live Spaces é seu espaço na internet com fotos (500 por mês), blog 
e agora com rede social http://spaces.live.com/


=
Instruções para entrar na lista, sair da lista e usar a lista em
http://www.mat.puc-rio.br/~nicolau/olimp/obm-l.html
=


[obm-l] [obm-l] Combinatória: número de soluções d e uma equação

2007-05-17 Por tôpico Pedro Cardoso

Saudações,

amigos da lista. Bem, surgiu aqui uma dúvida quando eu estava estudando 
combinatória. É em relação a uma variação não tão clássica do problema 
clássico do número de soluções inteiras não-negativas de uma equação.


x_1+x_2+x_3...+x_n = k

O número de soluções não-negativas e inteiras, para k também inteiro, é 
(k+n-1)/[k!*(n-1)!]. É fácil visualizar isso utlizando 'bolinhas' e 
'barrinhas'. Limitar por baixo o valor das incógnitas (garantir que todas 
ou algumas delas não possam ser inferiores a algum valor dado) também é 
simples. O problema é limitar 'por cima'. Exemplo:


x1+x2+x3+x4 = 21
x_i = 6, para qualquer i inteiro.

Como eu determino o número de soluções dessa equação?

Abraços,

Pedro Lazéra Cardoso

_
Descubra como mandar Torpedos SMS do seu Messenger para o celular dos seus 
amigos. http://mobile.msn.com/


=
Instruções para entrar na lista, sair da lista e usar a lista em
http://www.mat.puc-rio.br/~nicolau/olimp/obm-l.html
=


[obm-l] [obm-l] Divisão do preço de uma corrida de táxi

2007-05-09 Por tôpico Pedro Cardoso
Algo mui comum no cotidiano é rachar uma corrida de táxi quando o  caminho 
de uma pessoa é em parte comum com o caminho de um ou mais acompanhantes.


Bem, qual é a maneira mais justa de dividir o preço de uma corrida de táxi, 
considerando que esse preço já é conhecido desde o início pelos passageiros, 
se...


1- duas pessoas - a1 e a2 - querem ir a locais distintos, e o destino de a2 
fica no caminho de a1 (ou seja, a1, indo sozinho, passaria por onde a2 vai 
ficar)?

2- três pessoas - a1,a2 e a3 -... (caso análago ao anterior)?
3- n pessoas... (novamente análago)?
4- n pessoas, sendo que a trajetória sofre desfios porque a_i não passa por 
onde a_j ficaria?


Pedro Cardoso

_
Descubra como mandar Torpedos do Messenger para o celular! 
http://mobile.msn.com/


=
Instruções para entrar na lista, sair da lista e usar a lista em
http://www.mat.puc-rio.br/~nicolau/olimp/obm-l.html
=


[obm-l] Re: [obm-l] Ajuda (Combinatória)

2007-05-05 Por tôpico Pedro Cardoso

Eu acho que assim fica bem simples...

Total de maneiras, ignorando a imposição do problema: 3^8
Dessa forma eu contei três casos: todos os três números aparecem, dois dos 
três números aparecem, um dos três números aparece.


Agora vamos eliminar as que não valem:

1- só dois dos três números aparecem: (2^8)*3
(isso inclui só o 1 e o 2 aparecem, só o 2 o e 3 aparecem, só o 1 e o 3 
aparecem)


2- só um dos três números aparece: 3*1

S = 3^8 - 3*2^8 - 3 = 5790

_
O Windows Live Spaces é seu espaço na internet com fotos (500 por mês), blog 
e agora com rede social http://spaces.live.com/


=
Instruções para entrar na lista, sair da lista e usar a lista em
http://www.mat.puc-rio.br/~nicolau/olimp/obm-l.html
=


[obm-l] RE: [obm-l] Re: [obm-l] Ajuda (Combinatóri a)

2007-05-05 Por tôpico Pedro Cardoso
Opa, tem um erro: (1-) já inclui (2-) na solução do problema, e, além 
disso...


Só o 2 e o 3... contei  e 
Só o 2 e o 1... contei  e 
Só o 1 e o 3... contei  e 

Estou eliminando a mais. Então, S = 3^8 - 3*(2^8) + 3 = 5796

_
Verificador de Segurança do Windows Live OneCare: combata já vírus e outras 
ameaças! http://onecare.live.com/site/pt-br/default.htm


=
Instruções para entrar na lista, sair da lista e usar a lista em
http://www.mat.puc-rio.br/~nicolau/olimp/obm-l.html
=


[obm-l] Conjuntos - dúvida conceitual

2007-03-07 Por tôpico Pedro Cardoso

Olá amigos da lista,

estudando um pouco conjuntos fiquei com uma dúvida em relação ao conceito de 
CONJUNTOS DISJUNTOS.


Entendi que A e B são disjuntos se A(inter)B = vazio, mas, quando começo a 
trabalhar com três ou mais conjuntos...


Para N conjuntos serem disjuntos basta que a interseção simultãnea deles 
[quero dizer A(inter)B(inter)C...(inter)Z] = vazio? Ou é necessário também 
que eles sejam disjuntos dois a dois?


Se bastar que eles sejam disjuntos n a n [A(inter)B(inter)C...(inter)Z = 
vazio], não é sempre verdade que:


A,B,C...Z são disjuntos - n[A(união)B...(união)Z] = n(A)+n(B)+...+n(Z), 
certo?


Ex.: A ={1,2,3}; B ={2,4,6}; C={1,3,5}
A(inter)B(inter)C = vazio, mas [A(união)B(união)C] = {1,2,3,4,5,6} (6 
elementos)


e n(A)+n(B)+n(C) = 3+3+3 = 9.

Grato,

Pedro Lazéra Cardoso

_
Mande torpedos SMS do seu messenger para o celular dos seus amigos 
http://mobile.msn.com/


=
Instruções para entrar na lista, sair da lista e usar a lista em
http://www.mat.puc-rio.br/~nicolau/olimp/obm-l.html
=


[obm-l] RE: [obm-l] Soluções Inteiras

2006-11-24 Por tôpico Pedro Cardoso

Olá.

Quantas soluções inteiras tem a equação: x1 + x2 + x3 + x4 + x5 = 20 se 
cada xi é tal que xi é maior igual que 3 qualquer que seja o i pertencente 
a {1,2,3,4,5}?


Essa você resolve por combinatória, ivanzovski. Se x_i = 3, nós podemos 
reescrever o problema da seguinte forma:


x1 + x2 + x3 + x4 + x5 = 20 - 5.3 = 5, e agora x_i tem como única condição 
ser maior do que 0(depois você adiciona 3 a cada x_i).


Bem, o problema é explicar sem desenho. Fica (5+5-1)!!/[4!*(5-1)!] = 
9!/(4!5!) =  126.


Genericamente, se x_0 + x_1 + ... + x_i = n, existem (n+i-1)!/[n!*(i-1)!] 
soluções inteiras não negativas para a equação.


_
Inscreva-se no novo Windows Live Mail beta e seja um dos primeiros a testar 
as novidades-grátis. Saiba mais: 
http://www.ideas.live.com/programpage.aspx?versionId=5d21c51a-b161-4314-9b0e-4911fb2b2e6d


=
Instruções para entrar na lista, sair da lista e usar a lista em
http://www.mat.puc-rio.br/~nicolau/olimp/obm-l.html
=


RE: [obm-l] Soma infinita de arranjos

2006-11-23 Por tôpico Pedro Cardoso

Carlos Shine,

valeu pela solução. Mas não tem como provar o que você disse:
que 1/0! + 1/1! + 1/2! + ... +  1/n! se aproxima de 'e' para valores muito 
grandes de n?


Aproveito pra pedir desculpas ao Filipe, porque escrevi o nome dele errado.

Obrigado.

Pedro Lazéra Cardoso.

_
Experimente o novo Windows Live Messenger! 
http://get.live.com/messenger/overview


=
Instruções para entrar na lista, sair da lista e usar a lista em
http://www.mat.puc-rio.br/~nicolau/olimp/obm-l.html
=


RE: [obm-l] Soma infinita de arranjos

2006-11-18 Por tôpico Pedro Cardoso

Felipe,

valeu pela observação. Entendi o que você disse, e realmente o que eu queria 
era a resposta para uma soma finita de arranjos.


Como ainda não resolveram o problema, eu, numa última tentativa, repito ele 
aqui...


Quanto vale, em função de n,

A(n,0) + A(n,1)... + A(n,n-1) + A(n,n)?

Agradeço desde já.

Pedro Lazéra Cardoso

_
Chegou o Windows Live Spaces com rede social. Confira 
http://spaces.live.com/


=
Instruções para entrar na lista, sair da lista e usar a lista em
http://www.mat.puc-rio.br/~nicolau/olimp/obm-l.html
=


Re:[obm-l] lutas...

2006-11-15 Por tôpico Pedro Cardoso

Bem, eu acho que ficou assim...

Uma coisa que ajuda: o número mínimo de lutas ocorre quando um dado 
competidor não perde nenhuma disputa e todos os outros perdem duas. Seriam 0 
+ 2.199 = 398 derrotas = 398 disputas. O número mínimo de disputas então é 
398. Se todos os competidores já tiverem uma derrota, necessariamente 
teremos mais do que 398 lutas, necessariamente teremos 399 lutas.


Se entre duas lutas consecutivas de um mesmo competidor devem ocorrer lutas 
envolvendo outros dois competidores, ainda podemos ter as primeiras 200 
lutas. As lutas poderiam ser arranjadas assim:


(vencedor x perdedor)

001 002
003 004
005 006
... ...
199 200
--- (agora os pares ganham)
002 003
004 005
... ...
198 199
200 001

Bem, todos já têm uma derrota. Seria impossível termos 398 lutas, o que 
tornaria obrigatório termos 399 lutas. A questão é que, se um competidor não 
pode disputar duas lutas seguidas, não teremos um vencedor.


Suponha que a_n, a_p e a_k sobrem, cada um com uma derrota (essa 
configuração é obrigatória para que haja 399 lutas). Quaisquer que sejam os 
dois escolhidos para lutar, o vencedor dessa penúltima luta não poderá lutar 
novamente, porque entre as duas lutas dele deve ocorrer ao menos uma luta 
envolvendo ao menos outros dois compeditores. Não temos outros dois 
competidores.


a_n vence a_k. Sobram a_n e a_p, mas a_n não pode lutar duas vezes seguidas. 
Ficam os dois, cada um com uma derrota, impedidos de lutar.


_
Chegou o Windows Live Spaces com rede social. Confira 
http://spaces.live.com/


=
Instruções para entrar na lista, sair da lista e usar a lista em
http://www.mat.puc-rio.br/~nicolau/olimp/obm-l.html
=


[obm-l] Soma infinita de arranjos

2006-10-31 Por tôpico Pedro Cardoso

Olá.

Antes de qualquer coisa, queria pedir obrigado ao membro da lista que 
resolveu o problema sobre os jokers de um baralho, enviado por mim.


E agora um outro:

Quanto vale A(n,0) + A(n,1)... + A(n,n)?
Eu só vejo o pessoal resolvendo problemas parecidos com combinação, nunca 
com arranjo


Agradeço desde já.
Até.

Pedro Lazéra Cardoso

_
Seja um dos primeiros a testar o novo Windows Live Mail Beta- grátis. Acesse 
http://www.ideas.live.com/programpage.aspx?versionId=5d21c51a-b161-4314-9b0e-4911fb2b2e6d


=
Instruções para entrar na lista, sair da lista e usar a lista em
http://www.mat.puc-rio.br/~nicolau/olimp/obm-l.html
=


[obm-l] Jokers num baralho

2006-10-14 Por tôpico Pedro Cardoso
Aqui vai uma aparentemente bem fácil que está gerando um resultado pouco 
crível.


Num jogo de buraco, qual é chance que alguém tem de sair com uma mão que 
venha com ao menos um joker (coringa)?


Obs.: no buraco, cada jogador recebe inicialmente onze cartas, de um total 
de 56 cartas (contando com os jokers).


Pedro Lazéra Cardoso

_
Insta-le agora o Windows Live Messenger 
http://get.live.com/messenger/overview


=
Instruções para entrar na lista, sair da lista e usar a lista em
http://www.mat.puc-rio.br/~nicolau/olimp/obm-l.html
=


[obm-l] Re: [obm-l] Problema da média aritimética

2006-10-12 Por tôpico Pedro Cardoso

Acho que a solução é bem mais simples, Peter.

Se a média aritimética vale 16,1...

(2+n)/2 = 16,1 e (1 + (n-1) )/2 = 16,1

Daí se tira que n = 31 ou 32.

Mas (1+2+3+...+k-1)+(k+1+...+n)=16,1*(n-1). Logo, 16,1*(n-1) é inteiro.

16,1*(32-1) = 499,1
16,1*(31-1) = 483.

n = 31

Agora...

(1+2+3+...+k-1)+(k+1+...+n)=16,1(n-1)
(1+31).31/2 - k = 483
496 - k = 483 .:. k = 13


Resposta:

n = 31
k = 13

Pedro Lazéra Cardoso

_
Baixe agora o Windows Live Messenger! http://get.live.com/messenger/overview

=
Instruções para entrar na lista, sair da lista e usar a lista em
http://www.mat.puc-rio.br/~nicolau/olimp/obm-l.html
=


[obm-l] Fatoriais

2006-09-12 Por tôpico Pedro Cardoso

Olá, amigos da lista. Preciso da ajuda de vocês pra resolver essa daqui:

(n^2)!  (n!)^2
Quais são os valores de n que satisfazem?

Fica óbvio que, para n =1, (n^2)! = (n!)^2 = 1, e que, para n =2 (maior ou 
igual a 2), depois de fazer alguns testes,


(n^2)!  (n!)^2. Mas eu queria uma solução mais elegante, que não abusasse 
de testes. Enfim, uma prova.


Grato,

Pedro Cardoso.

_
Insta-le já o Windows Live Messenger. A nova geração do messenger. 
http://get.live.com/messenger/overview


=
Instruções para entrar na lista, sair da lista e usar a lista em
http://www.mat.puc-rio.br/~nicolau/olimp/obm-l.html
=


[obm-l] RE: [obm-l] Olimpíada Paulista 2005

2006-08-15 Por tôpico Pedro Cardoso

Boa tarde,

(a) Para que três times pontuem pouco, vamos supor que o primeiro colocado 
vence todos os jogos. Os outros times empatam entre si, atingindo assim 2 
pontos, enquanto o primeiro colocado terá 9. Note que, se houver, nos jogos 
entre os três últimos colocados, ao menos uma vitória, um time já terá 3 
pontos. A pontuação mínima portanto é 2 pontos, e o time se classifica por 
sorteio.


(b) Sejam A,B,C,D as pontuações dos quatro times. Para que todos os times 
ganhem bastante pontos, vou supor que 18 pontos (o máximo) foram disputados.


A+B+C+D = 18

Para dar mais pontos às outras equipes, vamos supor D = 0.

A+B+C = 18

Para que C, o terceiro colocado, seja o maior possível e ao mesmo tempo 
menor menor ou igual a A e a B, a = b = c = 6. O time com C pontos não se 
classifica no sorteio. Note que, se c  6, a+b  12 ... a  6 ou b  6, e 
então C se classificaria (o que não pode acontecer).


Acho que é isso.
Boa sorte para a sua filha.

_
Baixe agora o Windows Live Messenger! http://get.live.com/messenger/overview

=
Instruções para entrar na lista, sair da lista e usar a lista em
http://www.mat.puc-rio.br/~nicolau/olimp/obm-l.html
=


[obm-l] RE: [obm-l] Segunda Fase, Nível 1, Parte B da XXVII OBM

2006-07-27 Por tôpico Pedro Cardoso
Problema: Considere tres numeros inteiros positivos consecutivos de tres 
algarismos tais que o menor e multiplo de 7, o seguinte e multiplo de 9 e o 
maior de 11. Escreva todas as sequencias de numeros que satisfazem essas 
propriedades. 


É bom adiantar que a solução do Carlos Shine é bem mais eficiente por ser 
mais curta, mas eu mesmo assim mando a minha.


Sejam n, n+1 e n+2 os números dessa sequência.
n = 7a; n+1 = 9b; n+2 = 11c

7a+1 = 9b = 9b - 7a = 1.  Um par que é solução dessa equação é {4;5}.
Genericamente, a = 5 + 9k; b = 4 + 7k.
(qualquer valor de 'a' gerado por 5+9k gera um múltiplo de 7 cujo sucessor é 
múltiplo de 9)


7a+2 = 11c = 11c -7a = 2. Um par que é solução dessa equação é {4;6}.
Genericamente, a = 6 + 11m; b = 4 + 7m.
(qualquer valor de 'a' gerado por 6+11m gera um múltiplo de 7 cujo sucessor 
do sucessor (7a+2) é múltiplo de 11)


Daí tiramos que a = 5+9k = 6+11m = 9k - 11m = 1. O par {5;4} é solução, e, 
genericamente...

k = 5 + 11j; m = 4 + 7j.

Substituindo k em a = 5+9k, temos a = 5 + 9(5+11j) = 50 + 99j.
Como n = 7a, n = 350 + 693j. A única solução entre 100 e 1000 (ou seja, com 
números de três algarismos) é n = 350, pois, para c = 1, temos n = 1043 (e a 
sequência seria 1043, 1044, 1045). Então, a única sequência que satisfaz 
essa propriedade é 350, 351, 352.


Pedro Lazéra Cardoso

_
Acompanhe os desfiles do evento São Paulo Fashion Week. !

=
Instruções para entrar na lista, sair da lista e usar a lista em
http://www.mat.puc-rio.br/~nicolau/olimp/obm-l.html
=


RE: [obm-l] Desigualdade

2006-07-24 Por tôpico Pedro Cardoso

Olha, eu fiz assim:

x^2 + 4y^2 + 2z^2 + 4xy = (2y+x)^2 + 2z^2 =  (2y+x)^2 + (z.2^0,5)^2
Como a^2 + b^2 = (a+b)^2 - 2ab, fazendo a = 2y+x, b = z.2^0,5,

(2y+x)^2 + (z.2^0,5)^2 = [ 2y + x + z.2^0,5]^2 - 2yxz.2^0,5 = (2y + x + 
z.2^0,5)^2 - 64.2^0,5


Bom, vemos aí que y influencia o resultado mais do que z, que por sua vez 
influencia o resultado mais do x. Logo, xzy, com x = 2y = z.2^0,5. Como 
são inteiros, eu não tenho como precisar x em função z, mas sei que x = 2y. 
Agora vou voar um pouco, sem ter certeza se estou certo: como x  z, mas z 
mais próximo de x do que 2z, x = z (só concluo isso avaliando so possíveis 
valores de x,y,z e concluindo que, se duas das incógnitas não são iguais, 
uma é, no mínimo, o dobro da outra).


Daí, xyz = 32, x = 2y = z = y = x/2; z = x; x^3 = 64 = x = 4

y = 2; z = 4

E o menor valor possível é 4^2 + 4.2^2 + 2.4^2 + 4.2.4 = 96.

Olha, eu viajei um pouco, fiz algumas suposições, mas acho que tá certo. 
Tomara.


Pedro Lazéra Cardoso.

_
Seja um dos primeiros a testar o novo Windows Live Mail Beta- grátis. Acesse 
http://www.ideas.live.com/programpage.aspx?versionId=5d21c51a-b161-4314-9b0e-4911fb2b2e6d


=
Instruções para entrar na lista, sair da lista e usar a lista em
http://www.mat.puc-rio.br/~nicolau/olimp/obm-l.html
=


[obm-l] Sobre a OBM e o número de participantes

2006-07-24 Por tôpico Pedro Cardoso

Saudãções.

Tenho algumas dúvidas em relação ao número de participantes da olimpíada 
brasileira de matemática. Quantos alunso fazem as provas da primeira fase 
dos níveis 1,2 e 3?  Qual é a porcentagem de alunos que, em geral, passa à 
fase seguinte (li num link recomendado pelo próprio site da OBM, 
teorema.mat.br, que quase sempre cerca de 10% dos participantes avançam à 
segunda fase)?


Agradeço a quem me esclarecê-las,

Pedro Lazéra Cardoso.

_
Descubra aqui como mandar Torpedos Messenger! 
http://www.msn.com.br/artigos/maguire/default.asp 
http://www.msn.com.br/artigos/maguire/default.asp


=
Instruções para entrar na lista, sair da lista e usar a lista em
http://www.mat.puc-rio.br/~nicolau/olimp/obm-l.html
=


[obm-l] Re:[obm-l] FORMULAS DE REDUÇÃO

2006-07-09 Por tôpico Pedro Cardoso
Caro Cláudio, o trecho de poema abaixo, minha terra tem palmeiras/onde 
canta o sabiá, não pertence a Lusíadas e nem a nenhum outro poema de 
Camões. Esses versos são de Gonçalvez Dias, em Canto do Exílio.


sen(a+b) usa os primeiros 2 versos dos Lusiadas, de Camoes:
Minha terra tem palmeiras
Onde canta o sabia
seno a cosseno b
seno b cosseno a.

Depois eh soh lembrar que:
cos(-a) = cos(a);
sen(-a) = -sen(a);
cos(a) = sen(90 - a);
a = (a+b)/2 + (a-b)/2;
b = (a+b)/2 - (a-b)/2

Abraços!

Pedro Cardoso

_
Inscreva-se no novo Windows Live Mail beta e seja um dos primeiros a testar 
as novidades-grátis. Saiba mais: 
http://www.ideas.live.com/programpage.aspx?versionId=5d21c51a-b161-4314-9b0e-4911fb2b2e6d


=
Instruções para entrar na lista, sair da lista e usar a lista em
http://www.mat.puc-rio.br/~nicolau/olimp/obm-l.html
=


[obm-l] Probabilidade - Rolagem de dados

2006-07-09 Por tôpico Pedro Cardoso
Se uma pessoa lança x dados de y faces (numeradas de 1 a y), qual é a chance 
que ela tem de obter um certo resultado z na soma dos valores obtidos em 
cada rolagem?


Eu me propus esse problema e não consegui achar uma solução geral - apenas 
uma específica para cada valor de x, que, pelo que observei, é expressa por 
uma fórmula de grau x-1. Outro detalhe: essa fórmula só funciona até o valor 
médio (Vm) da soma, que é sempre o mais provável.


Para x =2 e y = 6, por exemplo, Vm = (2.1 + 2.6)/2 = 7.
Genericamente, Vm = (x + yx)/2 = x(y+1)/2

Os valores acima de x(y+1)/2 têm chance de ocorrência igual ao número que é 
tão distante de Vm quanto ele.  Para x = 2 e y = 6 (Vm = 7), a chance da 
soma 9 é igual à chance da soma 5. Genericamente, chamando chance de rolagem 
da soma n de C(n),
C(a) para a  x(y+1)/2 = C(x(y+1) - a), o que equivale a dizer que, se a+b = 
x(y+1), C(a) = C(b).


Bem, mesmo que eu tenha falado só besteira - esse é um medo que tenho! - 
ainda fica a questão, proposta no início do e-mail, e meus agradecimentos 
pela atenção.


Pedro Lazéra Cardoso

_
Você sabe em qual Copa o Pelé vestiu a camisa 10 pela 1a. vez? 
http://copa.br.msn.com/extra/curiosidades/1958/


=
Instruções para entrar na lista, sair da lista e usar a lista em
http://www.mat.puc-rio.br/~nicolau/olimp/obm-l.html
=